Bild: Pixabay, gemeinfrei.

Raumfahrt ist normalerweise extrem teuer, unter anderem wegen der großen Energien, die man braucht, um die Nutzlast auf hohe Geschwindigkeiten zu beschleunigen. Manchmal kann die Energie allerdings auch kostenlos sein: Bei Swing-by-Manövern, bei denen man ein Raumfahrzeug an einem Planeten vorbeifliegen lässt und es von dessen Schwerkraft mitschleppen und beschleunigen lässt. Damit kann man leider keine Geschwindigkeiten erreichen, wie sie für die Reise zu anderen Sternen nötig wären. David Kipping hat sich nun kürzlich eine Methode überlegt, wie dies dennoch möglich sein sollte – zumindest theoretisch.

 

Kostenlos Energie tanken – so geht’s

Swing-by-Manöver sind eine seit den Zeiten der Voyager-Raumsonden bewährte Technik, mit der sich Raumsonden Energie bei Vorbeiflügen an Planeten stibitzen. Das funktioniert folgendermaßen: wenn sich die Raumsonde nähert, wird sie vom Planeten angezogen und dadurch beschleunigt und abgelenkt. Entfernt sie sich wieder vom Planeten, so bremst dieser sie wieder ab. Aus seiner Sicht gesehen verlässt sie ihn am Ende mit derselben Geschwindigkeit, mit der sie sich ihm zu Beginn genähert hat, nur in einer anderen Richtung. Aus Sicht der Sonne hat sich jedoch zu der Geschwindigkeit der Sonde die des Planeten addiert und sie ist relativ zur Sonne effektiv schneller geworden. Den Preis dafür zahlt der Planet, der einen winzigen Ticken langsamer geworden ist, aber weil seine Masse (denke “Jupiter”) 1024-mal so groß ist wie die der Raumsonde, wird er auch nur den 1024ten Teil langsamer als die Sonde schneller geworden ist, und das kann niemand messen.

Wenn eine Raumsonde sich von einem Planeten mitziehen lässt, dann gewinnt sie an Geschwindigkeit (Länge des roten Pfeils, Diagramm unten rechts). Bild: Wikimedia Commons, Y Tambe, CC BY-SA 3.0.

Das Prinzip ist ähnlich demjenigen, einen Ball gegen einen entgegen kommenden Lastwagen zu treten. Aus Sicht des Lastwagens kommt der Ball mit seiner Schussgeschwindigkeit, sagen wir mal 50 km/h, plus der Geschwindigkeit des LKWs, sagen wir 70 km/h, entgegen, das sind zusammen 120 km/h. Der Ball verlässt den LKW aus dessen Sicht dann auch wieder mit 120 km/h in der Gegenrichtung. Aus Sicht des Ballschützen wurde der Ball mit 50 km/h weggetreten und kommt nun mit 120 km/h relativ zum LKW plus dessen Geschwindigkeit von 70 km/h zurück, macht in Summe 190 km/h. Die Kollision mit dem LKW hat ihn also um satte 140 km/h schneller gemacht, das Doppelte der Geschwindigkeit des LKW.

Und das gilt auch für den Swing-by: man kann die Geschwindigkeit maximal um die doppelte Geschwindigkeit des Planeten vergrößern. Im Sonnensystem bewegen sich die Planeten mit Geschwindigkeiten zwischen 47,4 km/s (Merkur) und 5,4 km/s (Neptun) auf ihren Bahnen – auf einen nennenswerten Bruchteil der Lichtgeschwindigkeit (knapp 300.000 km/s), den man für eine interstellare Reise bräuchte, kommt man so – leider – nicht.

 

Eine bessere Swing-by-Schleuder

1963 hat Freeman Dyson (ja genau, der mit den Dyson-Sphären) vorgeschlagen, wie man durch Swing-by-Manöver (auf Englisch auch Slingshot-Manöver genannt) auf relevante Bruchteile der Lichtgeschwindigkeit kommen könnte. Alles, was man braucht, ist ein enger Doppel-Neutronenstern. Ich hatte mal über den Doppel-Neutronenstern PSR J1946+2052 geschrieben, der engste bisher entdeckte, bei dem sich die Komponenten in einem Abstand von 346.000 km auf einer fast kreisförmigen Bahn alle 1h53m umkreisen  – das entspricht einer Bahngeschwindigkeit von rund 160 km/s – das ist noch nicht sooo berauschend. Dyson dachte eher an so etwas wie die Quelle des LIGO-Gravitationswellenereignisses GW1708017: zwei Neutronensterne von jeweils ca. einer Sonnenmasse, die sich alle 5 Millisekunden einmal umkreisen – in diesem Fall läge die Orbitalgeschwindigkeit bei 40.500 km/s und ein Raumschiff könnte bei einem engen Vorbeiflug auf bis zu 81.000 km/s beschleunigt werden. Das sind immerhin 27% der Lichtgeschwindigkeit. Nachteil: die Gezeitenkraft in der Nähe des Doppelneutronensterns wäre gewaltig und würde das Raumschiff vermutlich spaghettifizieren (inklusive Bolognese durch die Crew). Nicht nur Schwarze Löcher können so was.

 

Die ultimative Schleuder

David M. Kipping (den wir schon kennengelernt haben) hat nun eine Idee ersonnen, wie man noch schneller werden könnte – ohne spaghettifiziert zu werden! Hierfür wählt man statt eines engen Neutronen-Doppelsterns ein enges Paar Schwarzer Löcher. Die Grundidee ist die folgende: Man kann Schwarze Löcher als Spiegel für einen Laser verwenden. Man sendet Laserlicht zu demjenigen Schwarzen Loch, das einem gerade entgegen kommt. Aber man sendet das Licht selbstverständlich nicht in das Schwarze Loch hinein – dann kehrt es nicht zurück, wie der Name des Objekts schon nahe legt. Nein, man lenkt das Licht sehr gezielt haarscharf am Schwarzen Loch vorbei. Dieses ist nämlich von einem sogenannten “Photonen-Halo” umgeben, einer Zone, in der Licht das Schwarze Loch ein- oder mehrfach umkreist. Licht, das dem Schwarzen Loch näher als anderthalb Schwarzschildradien kommt (dieser Wert gilt für nichtrotierende Schwarze Löcher), bleibt mindestens für einige Orbits im Photonen-Halo gefangen, aber Licht, das knapp außerhalb dieses Radius vorbei saust, wird um 180° umgelenkt und kehrt wie ein Bumerang zurück – da Licht sich auf sogenannten Null-Geodäten bewegt, das sind kürzeste Verbindungen durch die gekrümmte Raumzeit mit Eigenzeit 0, spricht man daher auch von Bumerang-Nullgeodäten.

Ein Laserstrahl, der von dem Raumschiff unten (source) im richtigen Abstand von ca. 3/2 Schwarzschildradien vorbei gelenkt wird, wird vom Schwarzen Loch um α≈180° umgelenkt und kehrt zur Quelle zurück (“Bumerang-Null-Geodäte”). Bewegt sich das Schwarze Loch mit der Geschwindigkeit βBH auf das im Abstand d voraus fliegende Raumschiff zu, dann gewinnt der Laserstrahl aus Sicht des Raumschiffs an Energie (die Frequenz steigt von νi auf νf > νi), solange das Schwarze Loch sich schneller als das Raumschiff (Geschwindigkeit β1) bewegt. Das Raumschiff wird von einem Laserphoton inklusive des Rückstoßes beim Aussenden auf die Geschwindigkeit β3 > β1 beschleunigt. Bild: David Kipping, [1], arXiv, gemeinfrei.

Das Licht kehrt allerdings mit extra Schmackes zurück, denn das Schwarze Loch bewegt sich mit einem signifikanten Teil der Lichtgeschwindigkeit auf das Raumschiff zu. Das Licht kann selbstverfreilich nicht schneller werden, aber seine Frequenz nimmt zu und damit auch sein Impuls und seine Energie. Wenn das Raumschiff ein Laser-Photon aussendet, bekommt es einen kleinen Rückstoß durch den Impuls des wegfliegenden Photons und noch einmal einen deutlich größeren, wenn das Photon mit höherer Frequenz zurückkommt und vom Raumschiff wieder aufgefangen wird. Die ebenfalls höhere Energie kann genutzt werden, um weitere Photonen zu erzeugen. So wird man also kostenlos vom Schwarzen Loch angeschoben – ähnlich wie beim Swing-by-Manöver an einem Doppel-Neutronenstern, nur in sicherem Abstand.

Man muss dem Schwarzen Loch nämlich nicht außerordentlich nahe kommen. Wie das folgende Bild nebst Bildunterschrift zeigt, ist der Schub der Photonen umso stärker in die gewünschte Richtung gerichtet, je weiter man sich vom Schwarzen Loch fernhält. Damit kann man die Spaghettifizierung vermeiden.

Der Winkel δ, um den man den Laserstrahl für eine Bumerang-Ablenkung am Schwarzen Loch vorbei lenken muss, wird mit zunehmender Entfernung d des Raumschiffs vom Schwarzen Loch kleiner. Links oben beträgt d zwei Schwarzschildradien und der Winkel δ fast 70°. Bei 5 Schwarzschildradien sind es nur noch 27° und bei 20 Schwarzschildradien 13,5°. Je mehr Abstand man vom Schwarzen Loch einhält, desto mehr Anteil des Photonenimpulses erhält in Richtung der gewünschten Beschleunigung. Das Diagramm unten zeigt, dass der Winkel δ ungefähr linear mit d fällt. Die Proportionalitätskonstante ist der Winkel δ0=268,4° (wenn d in Einheiten von halben Schwarzschildradien GM/c² angegeben wird). Die Verzerrungen des Schwarzen Lochs in den Bildern oben sind übrigens nur scheinbar und der Skalierung der Diagrammachsen geschuldet. Bild: David Kipping, [1], arXiv, gemeinfrei.

Allerdings kommt es mit zunehmender Entfernung zu fortschreitender Dispersion des Lichts, d.h. zum Aufweiten des Laserstrahls, da nicht jedes Photon exakt den gleichen Weg nehmen wird, sondern sie bei verschiedenen Minimalabständen zum Schwarzen Loch bei verschiedenen Ablenkwinkeln enden werden. Das Raumschiff müsste dann ein entsprechend größeres und schwereres Lichtsegel zum Auffangen der Photonen mitschleppen. Kipping hält dies aber für ein Ingenieursproblem, das man schon irgendwie in den Griff bekommen wird.

Interessant ist schließlich noch, wie schnell ein Raumschiff denn werden könnte. Kipping gibt dafür folgende Formel an:

βterm = 4/3 βBH – 28/81 β³BH + O5BH]

wobei βterm  die erreichbare Endgeschwindigkeit in Bruchteilen der Lichtgeschwindigkeit ist (v/c), βBH die Geschwindigkeit des Schwarzen Lochs, ebenfalls in Bruchteilen der Lichtgeschwindigkeit, und O 5BH] für “irgendwas in der Größenordnung von β5BH” steht. Da βBH < 1 ist, ist β5BH ≪ 1 und damit vernachlässigbar. Die erreichbare Endgeschwindigkeit liegt demnach ungefähr bei 133% der Geschwindigkeit des Schwarzen Lochs. Für βBH = 0,5 (also 50% der Lichtgeschwindigkeit) kommt man so auf rund 62% c Endgeschwindigkeit und für βBH = 0,8 sogar auf 89% c. Das sind die Geschwindigkeiten, die man gerne erreichen möchte, damit die Zeitdilatation greift und die Flugzeit für die Besatzung erträglich bleibt.

Natürlich muss man dem Schwarzen Loch auch entkommen können – wenn man zu nahe dran ist, dann ist die Fluchtgeschwindigkeit von dort aus sehr groß und man verliert einen großen Teil der aufgenommenen Geschwindigkeit wieder. Kipping rechnet vor, dass man bei einem Schwarzen Loch mit nur 5% der Lichtgeschwindigkeit schon einen Abstand von ein paar tausend Schwarzschildradien einhalten muss, damit es sich lohnt – wir erinnern uns daran, dass ein Schwarzschildradius ca. 3 km · Masse des Schwarzen Lochs in Sonnenmassen beträgt und beachten, dass Schwarze Löcher mindestens knapp 3 Sonnenmassen haben müssen, sonst wären es Neutronensterne; wir reden hier also über ca. 10.000 km Abstand. Bei einem Schwarzen Loch mit 20% Lichtgeschwindigkeit reichen hingegen schon 100 Schwarzschildradien oder um die 1000 km.

Bleibt das Problem, ob man ein binäres Schwarzen Loch, das sich hinreichend eng umkreist, in erreichbarer Nähe finden kann. Kipping ist da ganz optimistisch, es muss so um die 10 Millionen Schwarze Löcher in der Milchstraße geben, da kann das nächste ja nicht so weit weg sein. Kipping kann sich vorstellen, das ausreichend fortgeschrittene Zivilisationen ganze Netzwerke von interstellaren Autobahnen zwischen Schwarzen Löchern nutzen, um quer durch die Galaxie zu reisen, denn natürlich kann man sich vom Laserstrahl auch mit Hilfe eines Schwarzen Lochs am Ziel abbremsen lassen, wenn man ihm entgegen fliegt.

 

Was tun ohne Doppel-Black-Hole?

Schwarze Löcher sind natürlich nicht alle doppelt und selbst wenn, dann noch lange nicht so eng, dass sie sich mit halber Lichtgeschwindigkeit und schneller umkreisen, aber zum Glück rotieren Schwarze Löcher ja gerne mal mit annähernd c, und so ein rotierendes Kerr-Black-Hole beschleunigt ebenfalls Photonen auf Bumerang-Geodäten, ganz alleine selbst vermöge seiner Rotation und der mitrotierenden Raumzeit. Die notwendigen Gleichungen verspricht uns Kipping in einem zukünftigen Artikel zu lösen. 😉

Eine interessante Idee des Autors ist schließlich noch, dass ein Raumschiff nicht notwendigerweise seine eigene Laserlichtquelle sein muss, sondern wie beim Breakthrough-Starshot-Projekt könnte der Laser etwa auf einem Planeten montiert sein und man wählt die Ablenkung des Laserstrahls so, dass das Raumschiff von dem Black-Hole-geboosteten Laserstrahl getroffen und angeschoben wird. Eine Art Nachbrenner für den Laser. Dann kann man einen sehr viel größeren Laser verwenden und beim Raumschiff selbst wiederum sehr viel Gewicht sparen. Statt Licht könnte man übrigens auch Teilchenstrahlen (denke: “Ionenantrieb”) verwenden, für massive Teilchen gibt es ebenfalls Bumerang-Geodäten.

Wie man sieht, ist es tatsächlich – und sehr zu meinem Erstaunen – im Rahmen der Relativitätstheorie möglich, kostenlos signifikante Bruchteile der Lichtgeschwindigkeit zu erreichen, denn der Halo-Drive liefert mehr Photonenenergie zurück, als zu Beginn aufgewendet werden muss. Das lässt sich grundsätzlich beliebig hochskalieren: Theoretisch könnte man damit ganze Planetenmassen in Bewegung versetzen. Nur ein paar ingenieurstechnische Probleme sind auf dem Weg dahin noch zu lösen. Eines davon ist das Erreichen des nächsten geeigneten Schwarzen Lochs in vertretbarer Zeit. Dies sei den Lesern als Übungsaufgabe überlassen. 😉

 

Referenzen und weiterführende Quellen

[1] David Kipping, “The Halo Drive: Fuel-Free Relativistic Propulsion of Large Masses via Recycled Boomerang Photons”, 28. Februar 2019; arXiv:1903.03423.

[2] Matt Williams, “Using Black Holes to Conquer Space: The Halo Drive!“, Universe Today, 13. März 2019.

[3] David Kipping, “The Halo Drive“, Cool Worlds-Youtube-Channel, 01. März 2019.

 

Kommentare (123)

  1. #1 Dr. Webbaer
    3. April 2019

    Klang jetzt nicht so-o praktikabel, wie sieht es denn aus Sicht des hiesigen werten Inhaltegebers aus, wenn Sonden auf ihrer Fahrt sukzessive Energie irgendwie aufnehmen und in Bewegungsenergie umsetzen, bspw. sog. Sonnensegel meinend?

    Wäre dies die praktikabelste Fortbewegungsmöglichkeit von Raumsonden, wenn viel Zeit zur Verfügung steht?
    So dass extra-hohe Geschwindigkeit erreicht wird und sich an im Weltall vorhandenen Massen dann abgebremst wird.

    MFG
    Dr. Webbaer

  2. #2 bote19
    3. April 2019

    Die Masse eines Schwarzen Loches bestimmt sich doch auch aus seiner Gravitationsenergie. Wenn also jetzt eine ziemliche große Masse , z.B. ein Neutronenstern an diesem Schwarzen Loch vorbeiswingt, dann geht doch Gravitationsenergie als Bewegungsenergie auf den Neutronenstern über.
    Frage: Wenn jetzt das Schwarze Loch dabei seine kritische Masse verliert, die es zum Schwarzen Loch macht, dann bekommen wir einen Big Bang. ?

  3. #3 Alderamin
    3. April 2019

    @Dr. Webbaer

    Sonnensegel können eine tolle Sache sein, wenn sie richtig groß sind. In der englischsprachigen Wikipedia sind ein paar Zahlen genannt. 2 Jahre zum Jupiter, 8,5 zum Neptun.

    Man braucht dafür aber Segel von Quadratkilometer-Größe. Ich denke, man scheut noch vor der Herausforderung zurück, so was in Raumsonden zu verpacken und dann ausfalten zu müssen. Außerdem kann man damit nicht ohne weiteres am Ziel bremsen (Aerobreaking geht ggf.). Ich denke, vorläufig wird man eher auf Ionenantriebe setzen.

  4. #4 Dr. Webbaer
    3. April 2019

    Ich denke, vorläufig wird man eher auf Ionenantriebe setzen.

    Es geht hier sog. Stützmasse verloren, was diesem Antrieb die Nachhaltigkeit nehmen könnte.

    So dass Maßgabe i.p. intrinsischem Energiehaushalts von wie hier gemeinten Sonden darin bestehen müsste energie-autonom, auch für etwaig anfällige Reparaturarbeit, zu theoretisieren.

    MFG
    Dr. Webbaer (der gerne, auch in diesem Zusammenhang angibt, in etwa so, wie fast überall jeder Ausländer ist, nur Dilettant, interessierter Laie zu sein, allerdings durch SciFi, insbesondere auch durch technischen SciFi, insbesondere auch der 60er, sozusagen gestählt zu sein)

  5. #5 Ernst der Lage
    3. April 2019

    Mmmmmh… Raumschiff-Spaghetti mit Crew-Bolognese. Lecker!

  6. #6 JoselB
    3. April 2019

    @bote19: Insgesamt wird nur Bewegungsenergie übertragen. Man kann das leicht daran sehen, dass eine Umwandlung von Gravitationsenergie in Bewegungsenergie, die dann einem anderen Himmelskörper gehört, die Impulserhaltung verletzen würde.

    Etwas ausführlicher: Bei der Annäherung wird Gravitationsenergie aus der wechselseitigen Anziehung durch Beschleunigung von SL und NS zueinander in Bewegungsenergie umgewandelt. Aber erstens, wird diese Umwandlung beim Entfernen wieder umgekehrt, wodurch sich insgesamt nur die Verteilung der Bewegungsenergie verändert hat. Und zweitens stammt die umgewandelte Energie aus dem Gravitationsfeld zwischen SL und NS und nicht aus dem SL selbst.

    Man kann also auf diesem Weg dem SL selbst keine Masse entziehen. (Analog dazu sieht dass auch bei dem Antrieb mit Rotatationsenergie aus, man entzieht nur die Rotationsenergie. Das SL rotiert danach langsamer, wird aber nicht leichter)

  7. #7 Alderamin
    3. April 2019

    @bote19, Joselb

    Neben dem was Joselb schon sagte (es geht dem Schwarzen Loch keine Masse verloren):

    Frage: Wenn jetzt das Schwarze Loch dabei seine kritische Masse verliert

    Selbst wenn ein Schwarzes Loch durch Hawking-Strahlung Masse verliert, kann es beliebig leicht werden und bleibt trotzdem ein Schwarzes Loch. Es gibt eine Dichte oberhalb der eine Masse zum Schwarzen Loch kollabiert. Ist es kollabiert, dann ist die Dichte (theoretisch) unendlich groß und bleibt es auch. Da gibt’s keinen Weg zurück. Das Schwarze Loch kann dann nur noch durch Hawking-Strahlung “verdampfen”.

  8. #8 bote19
    3. April 2019

    Alderamin
    dass ein Schwarzes Loch immer ein Schwarzes Loch bleibt, (von der Hawkins Strahlung mal abgesehen) scheint mir nicht logisch.
    Und dass es eine Sigularität gibt, das finde ich auch nicht logisch.
    Sonst könnten ja überall “Minischwarzelöcher” existieren, die sich noch nicht einmal mit einem Gravitationsfeld umgeben können, weil sie ja keinen Raum beanspruchen. Und wenn r = 0, dann ist r² = 0
    und dann wäre( M1 x M2) / r ² = unendlich, was auch unmöglich ist.

  9. #9 Captain E.
    3. April 2019

    Das hatte neulich jemand (der hiesige Hausherr?) recht schön hingeschrieben, aber Fakt bleibt, dass von den zig Sonnenmassen, die ein Schwarzes Loch so auf die kosmische Waage bringt, die eine oder andere Sonnenmasse eben nicht jenseits des Ereignishorizonts auf ewig versenkt wurde, sondern auf dieser Seite der Grenze in der Raumzeit gespeichert ist. Diese Masse lässt sich teilweise wieder freisetzen, wie die per Gravitationswellendetektor beobachtete Verschmelzung von mittelschweren Schwarzen Löchern gezeigt hat.

    Ich habe aber leider immer keine Lösung dafür, wie man in akzeptabler Zeit zu so einem binären Schwarzen Loch gelangen könnte. 😉

  10. #10 Spinner
    am Schwarzen Loch 13
    3. April 2019

    Und mit einem Spiegel reflektiere ich die Laserstrahlung zurück ins Schwarze Loch auf die Umlaufbahn. Damit werden die Photonen bei jedem Umlauf Schwarzes Loch – Spiegel – Schwarzes Loch – Spiegel immer energiereicher und härter. Die Stützmasse wird quasi recycled.

    Fragt sich nur, wie man einen Spiegel für Gammastrahlen realisieren kann ? Denn nach genügend Umläufen sind das Gammastrahlen !

  11. #11 Alderamin
    3. April 2019

    @bote19

    dass ein Schwarzes Loch immer ein Schwarzes Loch bleibt, (von der Hawkins Strahlung mal abgesehen) scheint mir nicht logisch.

    Ist aber so. Wenn die Dichte hoch genug ist, kannst Du auch die Erde in ein Schwarzes Loch quetschen. Schwarzschildradius 1 cm oder so. Selbst Elementarteilchen könnten zu Schwarzen Löchern werden – davor hatten ja einige beim Einschalten des LHCs so große Angst.

    Und dass es eine Sigularität gibt, das finde ich auch nicht logisch.

    Finden viele Physiker auch nicht, aber wir haben noch keine Theorie, die was anderes sagt (Schleifen-Quantengravitation und Stringtheorie würden was anderes sagen, sind aber noch keine richtigen Theorien).

    Sonst könnten ja überall “Minischwarzelöcher” existieren,

    Es könnte sein, das solche beim Urknall entstanden sind und Dunkle Materie bilden. Man hat auch schon nach dem Hawking-Zerfall solcher SLs Ausschau gehalten, die könnten nämlich heute schon zerfallen und nicht erst in 1070 Jahren, wie die stellaren SLs.

    die sich noch nicht einmal mit einem Gravitationsfeld umgeben können, weil sie ja keinen Raum beanspruchen.

    In der ART steht nirgendwo, dass ein Objekt ein Volumen haben muss, um den Raum um sich herum zu krümmen. Eine Masse reicht völlig aus.

    Und wenn r = 0, dann ist r² = 0
    und dann wäre( M1 x M2) / r ² = unendlich, was auch unmöglich ist.

    r is hier nicht der Durchmesser des Objekts, sondern der Abstand zwischen anziehender Masse M1 und Probemasse M2. Und man rechnet da immer so, als ob es Punkte im Massenzentrum wären. Sonst müsste man über die Abstände jedes Paars von Atomen in M1 und M2 integrieren – wer will das schon.

    Übrigens nimmt die Schwerkraft innerhalb einer ausgedehnten Masse nach innen bis auf 0 ab. Nur die im Radius eingeschlossene Masse wirkt anziehend.

  12. #12 Till
    3. April 2019

    @Spinner

    Fragt sich nur, wie man einen Spiegel für Gammastrahlen realisieren kann?

    Für Gammastrahlen wird es auf jeden Fall schwierig. Für Röntgenstrahlung gibt es Lösungen, die werden sicherlich Teil des genannten Engineering Problems sein ;-).

  13. #13 bote19
    3. April 2019

    Alderamin
    dein Optimismus in Ehren, aber Masse ohne Raum, das geht nicht. Dann kannst du folgern, Energie braucht keinen Raum. Wenn du das akzeptiert, dann sind Masse, Raum und Zeit eingebettet in einer Kategorie , die wir Geist nennen. Geist unterliegt nicht den Kategorien Raum und Zeit, Geist ist ewig.

  14. #14 tomtoo
    3. April 2019

    @Alderamin #7

    Das ist spannend. Sagst du es gibt gravitationslose schwarze Löcher?

  15. #15 Karl-Heinz
    3. April 2019

    @Alderamin

    Übrigens nimmt die Schwerkraft innerhalb einer ausgedehnten Masse nach innen bis auf 0 ab. Nur die im Radius eingeschlossene Masse wirkt anziehend.

    Die Zeit vergeht in Nähe großer Massen langsamer. Wie sieht’s im obigen Fall aus?

  16. #16 JoselB
    3. April 2019

    @Karl-Heinz: Wenn ich das richtig in Erinnerung hab, dann werden sowohl die Zeitdiletation als auch die Raumverformung gegenüber einem unendlich entfernten Beobachter nicht aufgehoben, die Raumzeit ist aber lokal annähernd flach und damit gibt es keine Anziehung nach innen. Das heißt auch, dass innerhalb einer massereichen Hohlkugel der Zeitverlauf auch anders ist als außerhalb.

  17. #17 Karl-Heinz
    3. April 2019

    @JoselB

    Danke für die Antwort. Ich sehe es genauso.

  18. #18 Alderamin
    3. April 2019

    @bote19

    dein Optimismus in Ehren, aber Masse ohne Raum, das geht nicht.

    Wenn Du das sagst… Die Sache ist halt, die ART sagt voraus, dass die Masse in einem SL zum Punkt kollabiert und eine bessere Theorie haben wir noch nicht. Kann uns praktischerweise aber auch egal sein, weil außen drumherum ein Ereignishorizont gestülpt ist, der vor uns verbirgt, was wirklich innen drin steckt. Aussagen über die wahre Natur der Singularität sind derzeit reine Spekulation und meine Aussagen beziehen sich auf die ART, nicht auf das echte Zentrum eines Schwarzen Lochs, weil darüber niemand etwas sagen kann. Wir wissen es nicht.

    Dann kannst du folgern, Energie braucht keinen Raum.

    Definiere mal “Energie”. Kann man das anfassen?

    Wenn du das akzeptiert, dann sind Masse, Raum und Zeit eingebettet in einer Kategorie , die wir Geist nennen.

    Zusammenhang?

    Geist unterliegt nicht den Kategorien Raum und Zeit, Geist ist ewig.

    Sehe ich anders und tut außerdem überhaupt nichts zum Thema. Bitte nicht derailen!

  19. #19 Alderamin
    3. April 2019

    @JoselB

    So habe ich es auch verstanden.

  20. #20 Alderamin
    3. April 2019

    @tomtoo

    Sagst du es gibt gravitationslose schwarze Löcher?

    Nein? Wo soll ich das gesagt haben?

  21. #21 Stefan K.
    Laufenburg
    3. April 2019

    Wieder mal ein reizwoller Artikel, danke!

    Wenn es diesen Halo-Drive und diese von Kipping postulierten “Autobahnen” gäbe, frage ich mich dann schon, wo der nächste Zubringer ist.

    Oder ob wir zufällig gerade in einem unerschlossenen Hinterland unserer Galaxie leben, wo es noch nicht mal Landstrassen gibt.
    In No Mans Sky gibt es ja auch solche Gebiete 🙂

    Oder (so ungern ich das glauben möchte) ob das Fermi-Paradoxon mit den bisher nicht beobachteten Aliens, die das Halo-Drive angeblich ausgiebig nutzen, sogar noch Bestätigung bekommt.

  22. #22 Alderamin
    3. April 2019

    @Stefan K.

    Vielleicht liegen wir zu weit ab vom nächsten Zubringer. 🙂
    Ich glaube auch nicht dran, genau so wenig an Dyson-Sphären. Ich sehe Kippings Arbeit mehr im Zusammenhang mit “schaut mal was die ART hergibt”, eine praktische Umsetzung davon scheitert ja schon daran, dass wir nicht mal Proxima Centauri erreichen können, und mit Licht ein massives Raumschiff anzutreiben und auf ein paar 10000 km Strecke auf x% c zu beschleunigen ist nicht nur ein technisches Problem – selbst wenn das ginge, ohne das Raumschiff zu verdampfen, wären die G-s mit Sicherheit tödlich.

  23. #23 Zhar
    3. April 2019

    Na, vielleicht ist es auch gut, dass wir etwas weiter von einer solchen “Autobahn” weg sind, könnt ja leicht passieren, dass unser Planet unglücklicher weise in eine solche Verbindungslinie hineindriftet und dann kommt die interstellare Wirtschaft auf die Rechnung, dass die ganzen Umwege fliegen viel zu teuer sei und eine Sprengung dagegen doch recht günstig zu haben ist.. oh, und mit einer solchen Technik ist ein Planetenkiller, sagen wir ein hinreichend großes Raumschiff mit viel übrigen Geschenken der letzten Weihnacht, die eh keiner wirklich wollte, welches mit annähernd Lichtgeschwindigkeit auf ein Hallo vorbeikommt, doch auch realisierbar.

  24. #24 Karl derKaefer
    3. April 2019

    Viele Jahre lang war ich daran gescheitert, den SwingByEffekt so zu verstehen, dass ich ihn auch erklären kann. Und jetzt lässt mich Alderamin den Ball gegen den LKW treten. Wieso erst jetzt?
    Trotzdem vielen Dank dafür und auch für all die anderenBBeiträge und Kommentare!

  25. #25 Torq
    3. April 2019

    Den Artikel finde ich sehr gut. Die beste Erklärung von Swing-by, die ich je gelesen habe. Vielen Dank, Alderamin!

    Wenn Du das sagst… Die Sache ist halt, die ART sagt voraus, dass die Masse in einem SL zum Punkt kollabiert und eine bessere Theorie haben wir noch nicht.

    Eigentlich haben wir nicht nur keine bessere Theorie, sondern gar keine. Konsens ist doch eigentlich, dass die ART bei sehr kleinen Längenskalen ihre Gültigkeit verliert. Oder doch nicht?!

    (Außerdem sagt die ART nicht in allen Fällen voraus, dass die Masse in SL zu einem Punkt kollabiert. Da es keine statischen SL gibt, müsstest du m.E. zumindest von einer Ringsingularität ausgehen, sofern du mit der ART argumentierst. Macht aber wahrscheinlich keinen Unterschied für die Argumentation…)

    Kann uns praktischerweise aber auch egal sein, weil außen drumherum ein Ereignishorizont gestülpt ist, der vor uns verbirgt, was wirklich innen drin steckt.

    So die allgemeine Hoffnung. Gilt natürlich nicht mehr, falls irgendwer über eine nackte Singularität stolpern sollte…

    Aussagen über die wahre Natur der Singularität sind derzeit reine Spekulation und meine Aussagen beziehen sich auf die ART

    Ok, akzeptiert. Ob die Schlüsse dann aber valide sind? Siehe oben…

    nicht auf das echte Zentrum eines Schwarzen Lochs, weil darüber niemand etwas sagen kann. Wir wissen es nicht.

    Stimmt natürlich auch. Aber es gibt verschiedene Theorien (oder Thesen?), die mir plausibler erscheinen als eine Singularität. Zum Beispiel wird darüber spekuliert, dass bei der Plancklänge Schluss ist. D.h. alles müsste sich da reinquetschen. Und dann würde sie doch plötzlich wieder passen, die Aussage:

    dein Optimismus in Ehren, aber Masse ohne Raum, das geht nicht.

  26. #26 Alderamin
    3. April 2019

    @myself

    … ein rotierendes Kerr-Black-Hole beschleunigt ebenfalls Photonen auf Bumerang-Geodäten, ganz alleine selbst vermöge seiner Rotation und der mitrotierenden Raumzeit. Die notwendigen Gleichungen verspricht uns Kipping in einem zukünftigen Artikel zu lösen.

    Offenbar ist er schon dabei. Schöne Grafik dort:

    https://twitter.com/david_kipping/status/1113495919184424960

  27. #27 Alderamin
    3. April 2019

    @Torq

    Eigentlich haben wir nicht nur keine bessere Theorie, sondern gar keine. Konsens ist doch eigentlich, dass die ART bei sehr kleinen Längenskalen ihre Gültigkeit verliert. Oder doch nicht?!

    Wir haben eine Theorie, die Schwarze Löcher voraussagt und bis fast zum Ereignishorizont gut funktionieren müsste (mehr dazu: nächsten Mittwoch, 15:00…). Für das Innere des Schwarzen Lochs sagt sie die Singulrität voraus, und die ist vermutlich nicht in der Form gegeben. Damit ist es eine Theorie Schwarzer Löcher, die wahrscheinlich nicht komplett ist. Dazu “keine Theorie” zu sagen, finde ich ein bisschen hart.

    Außerdem sagt die ART nicht in allen Fällen voraus, dass die Masse in SL zu einem Punkt kollabiert.

    Weiß ich. Unser Himmelsbote sprach aber speziell von der Schwarzschildlösung.

    Kann uns praktischerweise aber auch egal sein, weil außen drumherum ein Ereignishorizont gestülpt ist, der vor uns verbirgt, was wirklich innen drin steckt.

    So die allgemeine Hoffnung. Gilt natürlich nicht mehr, falls irgendwer über eine nackte Singularität stolpern sollte…

    Die ja nach allgemeiner Ansicht verboten sein soll. Gleicher Link wie zuvor.

    Aussagen über die wahre Natur der Singularität sind derzeit reine Spekulation und meine Aussagen beziehen sich auf die ART

    Ok, akzeptiert. Ob die Schlüsse dann aber valide sind? Siehe oben…

    Welche Schlüsse? Ich sage nicht, dass im Inneren eines Schwarzen Lochs eine Singularität steckt (ich sage eigentlich fast immer, dass das wahrscheinlich nicht so ist). Ich weise lediglich darauf hin, dass die ART eine Singularität voraussagt. Und alles andere ist Spekulation.

    Stimmt natürlich auch. Aber es gibt verschiedene Theorien (oder Thesen?), die mir plausibler erscheinen als eine Singularität. Zum Beispiel wird darüber spekuliert, dass bei der Plancklänge Schluss ist.

    Ja, aber man weiß es halt nicht. Man kann sich doch auch einfach mal mit dem zufrieden geben, über das man ziemlich definitive Aussagen machen kann. Insbesondere wenn es weder für das Artikelthema noch für Bote19s Frage nach dem “Unterschreiten der kritischen Masse” irgendeinen Unterschied macht. Ein Schwarzes Loch wird auch nicht wieder in einen Neutronenstern rücktransformiert, wenn sich im Zentrum ein Planckvolumen mit der Masse von weinger als 2,8 Sonnen befindet. Der Zug zum Schwarzen Loch ist endgültig abgefahren, nachdem die Masse ihren Schwarzschildradius unterschritten hat.

  28. #28 Torq
    4. April 2019

    Dazu “keine Theorie” zu sagen, finde ich ein bisschen hart.

    Habe ich auch nicht behauptet, zumindest nicht was große Skalen anbelangt. Ich habe von der Gültigkeit im Bereich sehr kleiner Skalen gesprochen, dazu gehört keines deiner Beispiele.

    Die ja nach allgemeiner Ansicht verboten sein soll. Gleicher Link wie zuvor.

    Ich weiß. Aber sie ergeben sich halt ebenfalls aus der ART. Die Kerr-Gleichungen hast du selbst erwähnt. Und sollte man sie nachweisen, würde das alles sowieso keine Rolle mehr spielen…

    Welche Schlüsse?

    Der Schluss, dass es Masse ohne Raum geben kann.

    Man kann sich doch auch einfach mal mit dem zufrieden geben, über das man ziemlich definitive Aussagen machen kann.

    Aber definitive Aussagen darüber, ob es Masse ohne Raum geben kann, kann man ja gerade nicht machen. Nur darauf wollte ich hinweisen.

    Insbesondere wenn es weder für das Artikelthema noch für Bote19s Frage nach dem “Unterschreiten der kritischen Masse” irgendeinen Unterschied macht.

    Also auf die Diskussion hast ursprünglich du dich eingelassen. Daher gehe ich zunächst einmal davon aus, dass sie aus deiner Sicht nicht unpassend ist. Das zum Artikelbezug.

    Und ja, für botes ursprüngliche Aussage macht das keinen Unterschied. Aber für seine Aussage bzgl. Masse und Raum und deine nachfolgende Antwort schon…

    Ein Schwarzes Loch wird auch nicht wieder in einen Neutronenstern rücktransformiert, wenn sich im Zentrum ein Planckvolumen mit der Masse von weinger als 2,8 Sonnen befindet. Der Zug zum Schwarzen Loch ist endgültig abgefahren, nachdem die Masse ihren Schwarzschildradius unterschritten hat.

    Sehe ich genauso.

    PS.: Freue mich auf nächsten Mittwoch, 15:00. 🙂

  29. #29 Captain E.
    4. April 2019

    Apropos Einstein und kleine Skalen: Auch wenn es eher die SRT und nicht die ART sein dürfte, so gibt es doch das Phänomen der “relativistischen Elektronen”, mit denen man erst den goldenen Schimmer von reinem Gold erklären kann. Die ursprüngliche Anwendung war es zumindest nicht, aber man ist darauf gestoßen, dass sich der Effekt genau so erklären lässt.

  30. #30 stone1
    4. April 2019

    Interessante Idee, bleibt das Problem erst mal in akzeptabler Zeit ein geeignetes schwarzes (Doppel) Loch zu erreichen. Wenn man aber bereits einen solchen Antrieb hat ist man auf diese Laser-Beschleunigung eh nicht mehr angewiesen – obwohl so eine Art gratis tanken natürlich auch nicht schlecht ist.

  31. #31 bote19
    4. April 2019

    Torq,
    Über die Beständigkeit der Schwarzen Löcher. Wenn wir mal davon ausgehen, dass die Atome in einem Schwarzen Loch in ihre Bestandteile , den Quarks zerlegt werden, dann passen in ein Schwarzes Loch wahrscheinlich 10 hoch 9 mal mehr Teilchen hinein als angenommen. Die Größe eine Quarks nehme ich mal mit 10 hoch -18 m an.
    Und wenn wir zweitens annehmen, das die Gravitation nicht durch diese Quarks hervorgerufen wird, sondern nur ein Effekt der Raumkrümmung ist, dann kann ein Schwarzes Loch nur existieren, wenn Quarks vorhanden sind. Fehlen die Quarks, fehlt der Raum, fehlt die Raumverkrümmung, fehlt die Gravitationskraft.
    Man könnte analog sagen, ein Schwarzes Loch ist nur ein Aggregatzustand von Energie (in der Form der Quarks).
    Deswegen sind Schwarze Löcher nicht beständig.

  32. #32 Spinner
    am Schwarzen Loch 13
    4. April 2019

    Ein Schwarzes Loch wird auch nicht wieder in einen Neutronenstern rücktransformiert, wenn sich im Zentrum ein Planckvolumen mit der Masse von weinger als 2,8 Sonnen befindet. Der Zug zum Schwarzen Loch ist endgültig abgefahren, nachdem die Masse ihren Schwarzschildradius unterschritten hat.

    Mal sehen ob das mit dem Zitat klappt, ich hab das so noch nie gemacht. Aber was ich dazu sagen wollte : Es ist doch schon erstaunlich, wie man hier mit nicht belegbaren Behauptungen um sich wirft. Vielleich sollte man doch sagen, daß das nur nach der derzeit gängigen Theorie, von der man genau weiß, daß sie so nicht stimmen kann, so ist !

  33. #33 UMa
    4. April 2019

    @Alderamin: Die Defokussierung des Lichtstrahls ist ein ernste Problem, wenn der Abstand wesentlich größer als der Schwarzschildradius ist. Das schenkt die Nutzbarkeit aus größere Entfernung doch stark ein, da dann der größte Teil nicht mehr das Raumschiff trifft.

  34. #34 Torq
    4. April 2019

    @bote19

    Über die Beständigkeit der Schwarzen Löcher. Wenn wir mal davon ausgehen, dass die Atome in einem Schwarzen Loch in ihre Bestandteile , den Quarks zerlegt werden, dann passen in ein Schwarzes Loch wahrscheinlich 10 hoch 9 mal mehr Teilchen hinein als angenommen. Die Größe eine Quarks nehme ich mal mit 10 hoch -18 m an.

    Da greife ich mal Alderamins Worte auf: Wenn du das sagst…

    Und wenn wir zweitens annehmen, das die Gravitation nicht durch diese Quarks hervorgerufen wird, sondern nur ein Effekt der Raumkrümmung ist, dann kann ein Schwarzes Loch nur existieren, wenn Quarks vorhanden sind. Fehlen die Quarks, fehlt der Raum, fehlt die Raumverkrümmung, fehlt die Gravitationskraft.

    Wie Alderamin schon geschrieben hat: Der springende Punkt ist die Dichte. Der Kollaps wird initial durch die Masse ausgelöst. Sobald die kritische Dichte erreicht ist, bleibt es beim SL. Egal, in welcher Form die Materia vorliegt. Irgendwann kann das SL dann ggf. verschwinden, durch die Hawking-Strahung. Das ist aber der einzige (mir bekannte) Weg…

  35. #35 Alderamin
    4. April 2019

    @bote19

    Wenn wir mal davon ausgehen, dass die Atome in einem Schwarzen Loch in ihre Bestandteile , den Quarks zerlegt werden

    … dann wäre es kein Schwarzes Loch, sondern ein Quarkstern. Die sind hypothetisch und würden ähnlich wie Neutronensterne aussehen. Sie wären dann aber keine Schwarzen Löcher.

    Es ist keine Kraft bekannt, die den Kollaps der Masse bei der Entstehung des Schwarzen Lochs noch irgendwo stabilisieren könnte, deswegen sagt die ART, die Masse fällt zum Punkt zusammen (oder zur Ringsingularität, wenn die Masse rotiert). Ob dann eine Quantengravitation noch irgendeine Schranke vorschiebt, wissen wir, wie gesagt, nicht.

    Und wenn wir zweitens annehmen, das die Gravitation nicht durch diese Quarks hervorgerufen wird, sondern nur ein Effekt der Raumkrümmung ist,

    Die ART kennt keine Gravitation, sie kennt nur Raumzeitkrümmung und Gravitation ist die Scheinkraft, die auftritt, wenn man ein Objekt daran hindert, seiner Geodäte durch die Raumzeit zu folgen. Raumkrümmung wird verursacht durch Masse, Energiedichte, Druck, Scherung und Energiestromdichten (Energie-Impuls-Tensor). Irgendwas davon musst Du im Schwarzen Loch haben, sonst hast Du keine Schwerkraft. Beim Schwarzschild-Fall dürfte das ausschließlich Energiedichte sein, beim Kerr-Fall zusätzlich Energiestromdichte, aber frag’ lieber Martin Bäker oder Niels.

    Deswegen sind Schwarze Löcher nicht beständig.

    Nach gängiger Theorie sind Schwarze Löcher deswegen nicht beständig weil sie Hawking-Strahlung aussenden. Allerdings leben sie 1070 bis 10100 Jahre, bevor sie zerfallen – im Augenblick wachsen sie sogar noch durch die Aufnahme von Strahlung (sie geben weniger Hawking-Strahlung ab, als schon alleine anr kosmischer Hintergrundstrahlung in sie hinein leuchtet).

  36. #36 Spinner
    am Schwarzen Loch 13
    4. April 2019

    @ Torq

    Der springende Punkt ist die Dichte

    Das ist wirklich der Fall. Man kann die notwendige Dichte, um eine Masse im Schwarzschildradius zu konzentrieren, ausrechnen. Ich habe so ein Diagramm mal gesehen. Für 1kg Masse kommt man auf eine notwendige Dichte in der Größenordnung von 10^80 kg/m^3. Kennt jemand so dichte Massen ? Bei einer Masse von 10^53 kg ist die notwendige Dichte nur bei 10^-26 kg/m^3.

    Die kosmologische Einbahnstraße ist das Schwarze Loch nur, weil wir es nicht besser wissen ! Aber warscheinlich ist das nicht.

  37. #37 bote19
    4. April 2019

    Torq,
    die Dichte ist das Verhältnis von Masse zum Volumen. Die Dichte ändert sich, wenn die Bedingungen sich ändern. Und die Dichte hängt auch ab von der Temperatur.
    Und wenn die Temperatur zu hoch wird, wird die Dichte wieder geringer, weil sich das Schwarze Loch wieder ausdehnt.
    Deine Behauptung, dass die Raumkrümmung nicht rückgängig gemacht werden kann, ist unlogisch. Der Raum kann sich verformen. Etwa durch die Annäherung eines zweiten Schwarzen Loches. Ob jetzt die Gesamtdichte der vereinigten Schwarzen Löcher größer oder kleiner wird, ist noch nicht entschieden.
    Über die Eigenschaften des Raumes wissen wir nämlich noch sehr wenig.

  38. #38 Alderamin
    4. April 2019

    @Spinner

    Ein Schwarzes Loch wird auch nicht wieder in einen Neutronenstern rücktransformiert, wenn sich im Zentrum ein Planckvolumen mit der Masse von weinger als 2,8 Sonnen befindet.

    Es ist doch schon erstaunlich, wie man hier mit nicht belegbaren Behauptungen um sich wirft. Vielleich sollte man doch sagen, daß das nur nach der derzeit gängigen Theorie, von der man genau weiß, daß sie so nicht stimmen kann, so ist !

    Also, bis zum Kollaps unterhalb des Schwarzschildradius ist die ART mit an Sicherheit grenzender Wahrscheinlichkeit richtig. Ganz sicher werden wir es vermutlich nächsten Mittwoch wissen, wenn wir voraussichtlich ein Bild des Schattens eines Schwarzen Lochs bekommen.

    Die ART ist so gut belegt, wie es eine physikalische Theorie nur sein kann (und mehr als “Theorien” kennt die Naturwissenschaft nicht – die Kugelgestalt der Erde ist auch eine naturwissenschaftliche Theorie), und die Quantenphysik ebenso. Es ist keine Kraft bekannt, die den Kollaps eines Sterns, der nicht mehr durch den Entartungsdruck der Neutronen stabilisiert wird, noch aufhalten kann, und ein Neutronenstern ist mit 10 km Radius schon nahe am Schwarzschildradius von rund 4,2 km, den ein Neutronenstern mindestens hat. Wenn es so eine Kraft für diese Größenordnung gäbe, wüssten wir davon aus Beschleunigerexperimenten.

    Vielleicht gibt es noch irgendwelche Kräfte auf weit höherem Niveau, die wir noch nicht kennen, aber die würden den Stern bestenfalls tief innerhalb des Schwarzschildradius stabilisieren. Und aus dem Schwarzschildradius kommt außer Hawking-Strahlung (die eigentlich knapp außerhalb entsteht) nichts mehr raus, das verbietet die ART. Und zwar der Teil der gut belegt ist.

  39. #39 Alderamin
    4. April 2019

    @Spinner

    Das ist wirklich der Fall. Man kann die notwendige Dichte, um eine Masse im Schwarzschildradius zu konzentrieren, ausrechnen.

    Der Schwarzschildradius ist r=2GM/c², das Volumen (mal naiv gerechnet) also 4/3 π r³ = 4/3 π (2G/c²)³ M³, d.h. es steigt mit der dritten Potenz der Masse. Das heißt wiederum, dass die mittlere Dichte innerhalb des Schwarzschild-Volumens mit dem Quadrat der Masse fällt. Das zentrale Schwarze Loch in M87 hat 6,6 Milliarden Sonnenmassen, sein Schwarzschildradius ist knapp 20 Milliarden km, sein Volumen 3,25*1040m³. 6,6 Milliarden Sonnenmassen sind knapp 2*1030kg. D.h., die Dichte, die die Materie im Fall des supermassereichen Lochs in M87 übertreffen muss, ist nur 0,4 kg/m³. Das ist 1/3 der Dichte von Luft bei Normaldruck (1013 mbar) und 20°C!

    Kennt jemand so dichte Massen ?

    Neutronensterne bringen es schon auf auf rund 1018 kg/m³, das weiß man aus Beobachtungen, und diese Dichte hat man auch im Atomkern, mit viel Platz zwischen den Quarks. Das geht sich schon aus, wenn die Quarks zusammengeschoben werden.

  40. #40 bote19
    4. April 2019

    Alderamin
    Was willst du mit der mittleren Dichte eines Schwarzen Loches von 0,4 Kg/m³ aussagen ?
    Das bedeutet doch, dass es am Schwarzschildradius ziemlich leer ist.

  41. #41 Alderamin
    4. April 2019

    @bote19

    Dass die Materie nicht dichter zu werden braucht, um zum Schwarzen Loch mit Ereignishorizont zu werden. Natürlich sagt die ART, dass sie dann noch weiter kollabiert und dichter wird. Der Schwarzschildradius ist am Ende im wesentlichen leerer Raum. Aber es reicht, diese von der Masse abhängige Dichte einmal zu erreichen, dann ist der Kollaps nicht mehr aufzuhalten (es reicht sogar weniger, 11/10 [Edit]9/8[/Edit] Schwarzschildradien oder so, hatte unser Karl-Heinz mal recherchiert).

  42. #42 Torq
    4. April 2019

    @bote19:

    Da du mich direkt angesprochen hast, auch wenn meine Antwort teilweise redundant zu Aleramins Ausführungen ist:

    die Dichte ist das Verhältnis von Masse zum Volumen. Die Dichte ändert sich, wenn die Bedingungen sich ändern. Und die Dichte hängt auch ab von der Temperatur.
    Und wenn die Temperatur zu hoch wird, wird die Dichte wieder geringer, weil sich das Schwarze Loch wieder ausdehnt.

    Nö, das wird nicht passieren. Noch mal kurz zur Entstehung eines SL (Korrekturen ausdrücklick willkommen): Materie sammelt sich und ballt sich zusammen. Dadurch erhöht sich der Druck, die Krümmung der Raumzeit vergrößert sich, es sammelt sich mehr Materie, und so weiter und so fort. Bis der Druck die Materie so sehr komprimiert, dass die Dichte einen kritischen Wert erreicht. Dann kollabiert die Materie vollständig. Zur Singularität, in ein Planckvolumen, ggf. in irgend etwas ganz anderes. Was hinter dem Ereignishorizont genau passiert, kann die ART nicht beschreiben. Aber dass die Raumzeit kollabiert und sich unendlich/extrem krümmt, ist klar. Sonst gäbe es ja keine SL…

    Soll heißen: Die Existenz des SL an sich hängt nicht mehr von der Dichte der Materie ab, sobald diese kollabiert ist. Wir kennen keinen Effekt, der die (praktisch) unendliche Krümmung der Raumzeit wieder zurücknehmen könnte. (Und das gilt, obwohl/auch wenn die ART bei sehr kleinen Skalen versagt.)

    Deine Behauptung, dass die Raumkrümmung nicht rückgängig gemacht werden kann, ist unlogisch. Der Raum kann sich verformen. Etwa durch die Annäherung eines zweiten Schwarzen Loches. Ob jetzt die Gesamtdichte der vereinigten Schwarzen Löcher größer oder kleiner wird, ist noch nicht entschieden. Über die Eigenschaften des Raumes wissen wir nämlich noch sehr wenig.

    Die Raumzeit kann sich verformen. Diese Verformung kann grundsätzlich auch rückgängig gemacht werden, das bestreite ich doch gar nicht. Wenn sich bspw. ein Stern weiterbewegt, dann geht natürlich auch die Verformung zurück. Ich verstehe jetzt alledings nicht, wie die Kollision zweier schwarzer Löcher dazu beitragen soll, dass die Verformung zurückgeht. Die Krümmung der Raumzeit ist bereits (praktisch) unendlich groß. Weder die Zunahme der Masse noch andere Effekte können daran etwas ändern, siehe oben.

  43. #43 Torq
    5. April 2019

    @Captain E.:

    Apropos Einstein und kleine Skalen: Auch wenn es eher die SRT und nicht die ART sein dürfte, so gibt es doch das Phänomen der “relativistischen Elektronen”, mit denen man erst den goldenen Schimmer von reinem Gold erklären kann. Die ursprüngliche Anwendung war es zumindest nicht, aber man ist darauf gestoßen, dass sich der Effekt genau so erklären lässt.

    Ja, auch der niedrige Schmelzpunkt von Quecksilber und viele andere Eigenschaften von Materialien. Elektronen sind sehr klein, dazu noch Elementarteilchen. Aber sie sind geradezu riesig, verglichen mit dem Planckvolumen… Trotzdem, über die Abgrenzung von klein zu sehr klein habe ich mir in diesem Zusammenhang anscheinend bisher zu wenige Gedanken gemacht… Danke für den Hinweis!

  44. #44 Karl-Heinz
    5. April 2019

    @Alderamin

    Ich habe im Torsten Fließbach nachgeguckt. Es ist der Faktor 9/8. Die Feldgleichungen der ART ergeben, dass für R → (9/8)*rS der zentrale Gravitationsdruck eines Sterns divergiert. 😉

    @Torq

    Was hinter dem Ereignishorizont genau passiert, kann die ART nicht beschreiben.

    Die Singularität bei r = rS ist eine reine Koordinatensingularität und die kann beseitigt werden, was aber bei r=0 nicht der Fall ist. Genau hier bricht die Beschreibung der ART zusammen. In diesem Breich treffen sich auch die ART und die Quantentheorie. Die Allgemeine Relativitätstheorie (ART) und die Quantentheorie lassen sich im Rahmen der bekannten physikalischen Gesetze nicht miteinander vereinbaren. Widersprüche treten damit erst bei unvorstellbar kleinen Abständen von 10^-33 cm zutage. 😉

  45. #45 bote19
    5. April 2019

    Torq, Alderamin
    vielen Dank für eure ausführlichen Erklärungen. Ohne das notwendige mathematische Rüstzeug kann ich da nicht weiter diskutieren. Deswegen singuliere ich mich.

  46. #46 Captain E.
    5. April 2019

    @Karl-Heinz:

    Tja, so dicht an der Plancklänge (1,616 · 10^−35, also gerade einmal ein Hundertstel) könnten merkwürdige Dinge geschehen. Eventuell ist die Lichtgeschwindigkeit nämlich gar nicht konstant, sondern stattdessen Werte wie eben die Plancklänge, und bei sehr, sehr kleinen Wellenlängen hat die Lichtgeschwindigkeit einen ganz anderen Wert. (Andernfalls müsste man nämlich konstatieren, dass die Plancklänge davon abhängt, wie schnell der messende Beobachter ist!) Das ist schwer bis (fast) unmöglich zu beobachten, würde aber eine winzige Modifikation der Relativitätstheorien erfordern. Das könnte vielleicht genau die entscheidende sein.

  47. #47 Captain E.
    5. April 2019

    @Torq :

    […]

    Die Raumzeit kann sich verformen. Diese Verformung kann grundsätzlich auch rückgängig gemacht werden, das bestreite ich doch gar nicht. Wenn sich bspw. ein Stern weiterbewegt, dann geht natürlich auch die Verformung zurück. Ich verstehe jetzt alledings nicht, wie die Kollision zweier schwarzer Löcher dazu beitragen soll, dass die Verformung zurückgeht. Die Krümmung der Raumzeit ist bereits (praktisch) unendlich groß. Weder die Zunahme der Masse noch andere Effekte können daran etwas ändern, siehe oben.

    Ich möchte an die mittels LIGO beobachteten Verschmelzungen von Schwarzen Löchern erinnern.

    Zwei schwarze Löcher von rund 29 und 36 Sonnenmassen kreisten umeinander und fusionierten zu einem Schwarzen Loch von 62 Sonnenmassen, 3 Sonnenmassen an Energie wurden in Form von Gravitationswellen abgestrahlt.

    Diese drei Sonnenmassen können sich logischerweise nicht hinter dem Ereignishorizont der beiden Ausgangsobjekte befunden haben. Stattdessen dürfte es die Masse von ungeheuren Energiemengen (zur Erinnerung: E=mc²) gewesen sein, die in der gekrümmten, verdrillten und sich mitdrehenden Raumzeit gespeichert waren. Bei der Neuanordnung der Raumzeit des fusionierten Gesamtlochs war diese Energie “übrig geblieben” und konnte daher abgestrahlt werden. Klingt komisch, scheint aber tatsächlich so zu sein. Und immerhin entspricht diese Energie 4,6 % der gesamten Masse der beiden ursprünglichen Schwarzen Löchern. Das ist nicht so wenig, finde ich.

  48. #48 Karl-Heinz
    5. April 2019

    @Captain E.

    Zu #46

    Ja genau. 😉

    Die entgegengesetzte Haltung ist, dass die ART nur eine effektive Niedrigenergietheorie ist, die sich aus einer grundsätzlicheren, bislang unbekannten Planck-Skalentheorie ergibt, deren Freiheitsgrade sich sehr von denen der ART oder Quantenfeldtheorie (QFT) unterscheiden. Bei dieser Betrachtungsweise wird angenommen, dass die ART und mit ihr die kontinuierliche Raumzeit aus dieser noch zu findenden Theorie im Limes großer Abstände in ähnlicher Weise „entstehen“, wie sich die makroskopische Physik aus der Quantenwelt der Atome und Moleküle ergibt.

  49. #49 Captain E.
    5. April 2019

    @Karl-Heinz:

    So sehr entgegengesetzt sind diese beiden Betrachtungsweisen womöglich gar nicht. Das werden wir aber erst dann wissen können, wenn es eine brauchbare Hypothese gibt, für die man Belege beibringen kann und die sich nicht mal so eben falsifizieren lässt.

  50. #50 Torq
    5. April 2019

    Diese drei Sonnenmassen können sich logischerweise nicht hinter dem Ereignishorizont der beiden Ausgangsobjekte befunden haben. Stattdessen dürfte es die Masse von ungeheuren Energiemengen (zur Erinnerung: E=mc²) gewesen sein, die in der gekrümmten, verdrillten und sich mitdrehenden Raumzeit gespeichert waren. Bei der Neuanordnung der Raumzeit des fusionierten Gesamtlochs war diese Energie “übrig geblieben” und konnte daher abgestrahlt werden. Klingt komisch, scheint aber tatsächlich so zu sein. Und immerhin entspricht diese Energie 4,6 % der gesamten Masse der beiden ursprünglichen Schwarzen Löchern. Das ist nicht so wenig, finde ich.

    Das ist ganz sicher nicht wenig. In Summe nimmt die Masse aber ggü. jedem einzelnen SL zu. Und die (praktisch) unendliche Dichte verändert sich so oder so nicht. Oder zumindest nie so sehr, dass die kritische Dichte wieder unterschritten wird. Und auch die (praktisch) unendliche Krümmung der Raumzeit bleibt (praktisch) unendlich. Und darum ging es bote19 doch, oder? (Sorry, dass ich rein qualitativ argumentiere. Bei einer Singularität ergibt sich das automatisch, bei einem Planckvolumen müsste man es eigentlich mal durchrechnen, bevor man es dreist behauptet / einfach „praktisch“ unendlich schreibt. Aber meine Mathematikvorlesungen habe ich irgendwie effektiv verdrängt..,;-))

  51. #51 Captain E.
    5. April 2019

    @Torq:

    Nun ja, aber besteht diese (praktisch) unendliche Dichte nicht am Ereignishorizont? In größerem Abstand dazu sieht das dann anders aus, aber offensichtlich steckt dort trotzdem soviel Energie, dass sie als deutlich wahrnehmbare Masse auftritt. Und die Raumkrümmung von zwei verschmelzenden Schwarzen Löchern enthält offensichtlich erheblich weniger Energie als die Summe der beiden Vorgänger.

    Etwas anderes hatte nicht behaupten wollen. Abgesehen von diesem schwer zu fassenden Effekt der Hawking-Strahlung tut sich da nichts mehr am und jenseits des Ereignishorizonts. Was weg ist, ist weg.

  52. #52 Alderamin
    5. April 2019

    David Kipping hat gestern Abend ein neues Video über den Halo-Drive auf seinem Cool-Worlds-Channel eingestellt. Hatte noch keine Zeit, es anzuschauen, soll aber wohl Fragen beantworten, die an ihn gerichtet wurden.

  53. #53 Karl-Heinz
    5. April 2019

    @Torq

    Bei einer Singularität ergibt sich das automatisch, bei einem Planckvolumen müsste man es eigentlich mal durchrechnen, bevor man es dreist behauptet / einfach „praktisch“ unendlich schreibt

    Das Durchrechnen bei der ART bringt leider nicht’s, denn nach der ART strebt die Materie auf den singulären Punkt r=0 zu. 😉

  54. #54 Alderamin
    5. April 2019

    @Captain E.

    Nun ja, aber besteht diese (praktisch) unendliche Dichte nicht am Ereignishorizont?

    Nein, da ist nur die Fluchtgeschwindigkeit >= c. Wie in #39 vorgerechnet kann die mittlere Dichte eines Schwarzen Lochs bei hinreichender Masse kleiner als die von Luft unter Normaldruck sein.

    In größerem Abstand dazu sieht das dann anders aus, aber offensichtlich steckt dort trotzdem soviel Energie, dass sie als deutlich wahrnehmbare Masse auftritt. Und die Raumkrümmung von zwei verschmelzenden Schwarzen Löchern enthält offensichtlich erheblich weniger Energie als die Summe der beiden Vorgänger.

    Die abgestrahlte Energie steckte in der potenziellen Energie der kreisenden Schwarzen Löcher und war nicht exakt lokalisierbar. Die SLs konnten nur dadurch enger zusammenrücken, dass sie Energie in Form von Gravitationswellen aussendeten. Keines der beiden SLs hat selbst Masse verloren, ihre Ereignishorizonte wurden nicht kleiner.

    Die Verformung der Raumzeit kann sich natürlich ändern, sie hängt von der eingeschlossenen Masse ab. Das Problem ist, dass die Masse aus einem Ereignishorizont nicht mehr heraus kommen kann (nur über Hawking-Strahlung). Deshalb kann sich ein Schwarzes Loch nicht zurück in was anderes transformieren. Es kann nur wachsen, nicht schrumpfen (außer durch Hawking-Strahlung).

  55. #55 Hans Zekl
    Kaltenkirchen
    5. April 2019

    Hallo Aldemarin,

    ich wirklich schöner Artikel. Aber wie das leider bei interstellaren Antrieben so ist, der Teufel steckt im Detail. Wie Kipping in seinem Artikel selbst schreibt, ist das ganze ein Gedankenexperiment unter idealisierten Bedingungen. Ganz wichtig ist der Schlussabsatz, wo er feststellt, dass man erst per interstellarem Flug zu einem passenden Abstand zu einem SL kommt (Ich habe den Artikel allerdings nur überflogen). Was er nicht berücksichtigt ist einmal, die Energie die er für den ersten Laserschuss braucht. Die muss er erst einmal aufbringen. Auch die Energien für die die weitere Beschleunigung muss aufgebracht werden. Außerdem sagt er nichts über die Beschleunigungen, die erreicht werden können, d.h. wie lange es dauert, bis das Raumschiff auf genügend Geschwindigkeit kommt.
    Alles in allem, eine interessante Idee, aber in meinen Augen nicht praktikabel.

  56. #56 Captain E.
    5. April 2019

    @Alderamin:

    Wahrscheinlich habe ich auch gar nicht verstanden, was “diese (praktisch) unendliche Dichte” eigentlich sein soll.

    Nur eben was die Krümmung der Raumzeit angeht, so können wir diese ausschließlich diesseits des Ereignishorizonts bestimmen. Dahinter ist Terra Incognita. Womöglich gibt es noch nicht einmal Raumzeit – wir wissen einfach nichts darüber. Klar ist aber: Die Masse, die den Ereignishorizont überschritten hat, eixistiert rein rechnerisch noch, beeinflusst uns aber nicht mehr – mangels Übertragung, denn es kommt ja nichts mehr hinaus. Die von ihr zuvor erzeugte Raumzeitkrümmung bleibt bestehen und hat natürlich noch einen Effekt.

    So, und bei der Verschmelzung zweier Schwarzer Löcher kommt natürlich auch nichts mehr heraus, aber logischerweise ist auch das irrelevant – es kommt halt immer noch nichts heraus. Nur die Dinge, die die gekrümmte und mitrotierende Raumzeit in der Umgebung dieser beiden Schwarzen Löcher mit sich selber anstellt, führt offensichtlich zu einer nicht unerheblichen Rekonfiguration. Die “Addition” der Krümmungsgeometrien führt zu einer neuen Krümmungsgeometrie, die anders ist als zuvor, und das vor allem energieärmer (als die Summe der Einzelteile). Die Energie wird ja freigesetzt und verbreitet sich per Gravitationswellen im Universum.

    So rein hypothetisch könnte man also die gesamte Energie, die per Krümmung in der Raumzeit um ein Schwarzes Loch steckt, freisetzen, ohne dieses in irgendeiner Weise zu verändern. Der Trick ist nur der, dass es bestimmt keinerlei denkbare Transformation gibt, mit der dies möglich wäre. Würden tausende Schwarzer Löcher nacheinander miteinander verschmelzen, es würde immer Masse in Form von abgestrahlten Gravitationswellen verloren gehen. Die Energie/Masse könnte aber niemals auf Null fallen, und die resultierenden Schwarzen Löcher hätten immer eine größere Masse als jedes der jeweils beteiligten Vorläufer-Löcher.

    Leute, ich muss mal los, um mein Gehirn zu entknoten! 🙂

  57. #57 Alderamin
    5. April 2019

    @Hans Zekl

    Danke!

    Was er nicht berücksichtigt ist einmal, die Energie die er für den ersten Laserschuss braucht. Die muss er erst einmal aufbringen.

    Rein hypothetisch kann man je beliebig klein anfangen und mit der wieder aufgefangenen Energie immer wieder neu schießen.

    Technisch gesehen ist das Ganze natürlich nicht umsetzbar. Man hat ja auch gar nicht die Zeit, beliebig oft zu schießen, wenn man wirklich Fahrt aufnehmen will. An keiner Stelle geht er auf die nötige Leistung ein, die der Laser haben muss, um das Raumschiff innerhalb von 100-1000 RS auf einen nennenswerten Bruchteil von c zu bringen und was dies für G-Kräfte implizieren würde.

    Alles in allem, eine interessante Idee, aber in meinen Augen nicht praktikabel.

    Sehe ich genau so.

  58. #58 Alderamin
    5. April 2019

    @Captain E.

    So rein hypothetisch könnte man also die gesamte Energie, die per Krümmung in der Raumzeit um ein Schwarzes Loch steckt, freisetzen, ohne dieses in irgendeiner Weise zu verändern.

    Welche Energie soll denn konkret in dem Feld stecken?

    Wenn es Kerr-SL ist – ja, dann kann es über den Penrose-Prozess Rotationsenergie abgeben, aber ein Schwarzschild-SL kann alleine nichts abstrahlen (außer Hawking-Strahlung), Du brauchst immer irgendeine Masse, die Du von Weitem hineinfallen lässt, und dann greifst Du auf potenzielle Energie zurück, die der Urknall für Dich geschaffen hat, der überhaupt erst für den Abstand gesorgt hat, aus dem Du das Objekt in das SL fallen lassen kannst. Das SL wird dadurch noch größer und kann bei der nächsten Masse noch mehr Energie freisetzen (z.B. als Strahlung der Akkretionsscheibe).

    Wie gesagt ist das nicht wirklich Energie aus dem Gravitationsfeld des SLs, sondern Energie vom Urknall, die als potenzielle Energie in den Abständen der Massen zueinander steckt.

  59. #59 Torq
    5. April 2019

    @Karl-Heinz

    Das Durchrechnen bei der ART bringt leider nicht’s, denn nach der ART strebt die Materie auf den singulären Punkt r=0 zu.

    Ok, ja, das muss natürlich so sein. Da hätte ich selbst drauf kommen können. Das ist ja eigentlich mein eigenes Argument, nur konsequent(er) zu Ende gedacht… 😉

    Wie gesagt ist das nicht wirklich Energie aus dem Gravitationsfeld des SLs, sondern Energie vom Urknall, die als potenzielle Energie in den Abständen der Massen zueinander steckt.

    Das unterstreicht mal wieder beeindruckend, weshalb du derjenige von uns beiden bist, der hier bloggt… 😉

    Danke für die Erläuterung. Ich hatte jetzt eher an die kinetische Energie gedacht…

    Und klar, einen Massenverlust kann es nicht geben. Es ist also auch nicht sinnvoll, überhaupt über eine Verringerung der Dichte nachzudenken.

  60. #60 Alderamin
    5. April 2019

    @Torq

    Ich hatte jetzt eher an die kinetische Energie gedacht…

    Die entsteht ja erst aus der potenziellen (deswegen heißt die so – ist halt nur potenziell Energie). Klar, abgestrahlt wird sie erst nach der Wandlung in kinetische, die dann in die Energie der Gravitationswelle gewandelt wird und sich davon macht.

    Ein Teil der kinetischen Energie endet übrigens in der Rotation des verschmolzenen SLs.

  61. #61 JoselB
    5. April 2019

    Jetzt hat Alderamin meine Frage schon beantwortet, bevor ich sie überhaupt stellen konnte, nämlich ob sich die abgestrahlte Energie nicht aus der potentiellen Energie statt der Masse ergibt.

    Das hat auch Analogien in der Chemie und Nuklearphysik. Bei der chemischen Verbrennung ist das Verbrennungsprodukt (marginal) leichter als die Ausgangsstoffe. Die freiwerdende Energie wird als thermische Strahlung (analog zu Gravitationswellen bei der SL-Vereinigung) abgegeben. Bei der Kernfusion gilt das Gleiche, wobei der Massenunterschied schon etwas größer ausfällt. Die Energie stammt (nur leicht vereinfacht) in beiden Fällen aus der Bindung statt aus der Masse selbst. Ich hoffe mal, dass der Vergleich nicht zu sehr hinkt!

  62. #62 Captain E.
    5. April 2019

    @Alderamin:

    Ich meine mich erinnern zu können, dass einst (vermutlich “drüben”) die Frage aufkam, wieso die Masse eines Schwarzen Lochs überhaupt eine Auswirkung haben könnte, wenn Gravitationswellen mit Lichtgeschwindigkeit laufen. Die Antwort war dann die, dass durch Gravitationswellen Änderung der Krümmung der Raumzeit übertragen werden. Das Innere eines Schwarzen Lochs beeinflusst also “unser” Universum gar nicht mehr.

    Davon ausgehend müsste es eigentlich so laufen: Die Masse, aus der ein Schwarzes Loch entstanden ist, hat der Raumzeit ihren Stempel aufgedrückt. Und das bleibt dann erst einmal so, solange nicht andere Massen darauf einwirken. Die im Schwarzen Loch verschwundene Materie kann es nicht sein, da keine Wirkung herauskommt. Folglich dürfte die gesamte Raumkrümmung eines Schwarzen Lochs, die man messen und beobachten kann und die man manchmal “Schwerkraft” nennt, nur noch der Schatten oder Stempelabdruck des Schwarzen Lochs sein. Von der Materie, die diese ursprüngliche Krümmung verursacht hat und die jetzt im Schwarzen Loch steckt, ist sie im Grunde völlig abgekoppelt.

    Ein “Glätten” oder “Entfalten” der Raumzeit findet natürlich nicht spontan statt, sondern nur unter Einwirkung anderen Massen und Raumkrümmungsgeometrien, und auch immer nur teilweise. Für mich steht damit aber fest: Die gesamte beobachtbare Masse eines Schwarzen Lochs steckt in der Raumkrümmung diesseits des Ereignishorizonts, und genau von dort stammt sie auch, wenn zwei Schwarze Löcher verschmelzen und ein paar Sonnenmassen an Energie abstrahlen.

    Oder noch einmal anders: Laut ART ist Schwerkraft eine geometrische Eigenschaft (Krümmung) des Raumes, die von Massen verursacht wird. Über den Mechanismus, wie dies geschehen könnte, sagt uns die ART leider nichts, sondern gibt nur die Berechnung an, wie sich das ausdrückt. Die Massen, die zum Schwarzen Loch werden oder in eines hineinfallen, verursachen ebenfalls eine Krümmung – aber dann sind sie weg und interagieren nicht mehr mit dem Universum außerhalb. Die Krümmung bleibt bestehen, obwohl sie von den ursprünglichen Massen mittlerweise abgekoppelt ist. Bei der Verschmelzung zweier Schwarzer Löcher reagieren die von ihnen dem Raum aufgeprägten Geometrien miteinander, und die neue gemeinsame Raumzeitkrümmung ist energieärmer als die Summe der einzelnen. Bei dem beobachteten Ereignis wurde also quasi die Raumzeit um das neue größere Schwarze Loch um den Absolutbetrag von drei Sonnenmassen “geglättet”.

  63. #63 Alderamin
    5. April 2019

    @Captain E.

    Klingt nicht verkehrt. Jetzt müssten Niels oder Martin was sagen. 😉

  64. #64 Captain E.
    5. April 2019

    @Alderamin:

    Apropos “Energie vom Urknall”: Damit hast du natürlich recht, denn logischerweise hat jedes einzelne Elementarteilchen mit einer Masse von Anfang an seine eigene persönliche Raumzeitkrümmung, und jedes Fitzelchen (= Quant) Energie ebenfalls. Immer wenn sich eine noch so kleine Masse bewegt, verändert das die Raumzeit, und das Zusammenballen erst recht.

  65. #65 JoselB
    5. April 2019

    @Captain E., Alderamin:
    Vielen Dank für die vielen Überlegungen. Nur dank ein bischen Nachdenken bin ich auf Fragen gestoßen, bei denen ich keinen Ansatz für Antworten habe. Sind daher auch hauptsächlich als Diskusionsanregung gedacht, auch wenn ich mich natürlich über Antworten freuen würde:

    Gravitationswellen (GW) vermitteln ja Änderungen im Graviationsfeld durch die Beschleunigung der beteiligten Massen. Sie entstehen außerhalb des Ereignishorizontes der SLs und wirken im Fall von binären SLs auch auf das jeweilig andere SL ein um es auf Bahn zu halten. Stimmt das soweit? Da GW durch die Beschleunigung von SLs verursacht werden aber auch die Beschleunigung der Partner-SLs verursachen, die wiederum GW auslösen, müsste sich die komplette Bewegung aus dem G-Feld um die SLs herum berechnen lassen, ohne die eigentliche Singularität zu berücksichtigen? Ansonsten müsste ja Information von hinter dem Ereignishorizont entkommen können. Gleichzeitig kommen die GW natürlich durch den Ereignishorizont und können auch die Singularität selbst beschleunigen, oder? Oder wird die Beschleunigung komplett durch das umgebende Feld vermittelt und wirkt sich nur indirekt auf die Singularität aus? Und falls nicht, wie können sich das G-Feld und die Singularität synchron zueinander bewegen, obwohl sie komplett verschiedene Bedinungen aufweisen? Fragen über Fragen…

  66. #66 Torq
    6. April 2019

    @JoselB

    Sie entstehen außerhalb des Ereignishorizontes der SLs und wirken im Fall von binären SLs auch auf das jeweilig andere SL ein um es auf Bahn zu halten. Stimmt das soweit?

    Binäre SL können z.B. in einem wechselwirkenden Doppelsternsystem entstehen. Zunächst umkreisen sich beiden SL, wie auch die ursprünglichen Sterne. Bei ihrer Bewegung um den gemeinsamen Schwerpunkt emittieren sie allerdings Energie in Form von Gravitationswellen, sodass sich ihr Abstand mit der Zeit verringert. Irgendwann kommt es dann zum Merger. Ohne weitere Effekte würde das natürlich seeeehr lange dauern, aber dennoch gilt: Gravitationswellen stabilisieren das System nicht, ganz im Gegenteil.

    Oder habe ich deine Aussage gänzlich falsch verstanden?

  67. #67 JoselB
    6. April 2019

    @Torq: Das die GW Energie aus einem binären SL-System heraustragen und so zum Merger führen ist mir bekannt. Wenn ich mich nicht irre, würden die SL ohne GW nicht in der Form umeinander kreisen, wie sie es tun. Aus sicht des ersten SL enthält die Raumzeit über das zweite SL nur die Information, dass da eine kompakte Masse ist, die sich bewegt. Ich bin mir nicht sicher ob auch die Information enthalten ist, dass diese Masse in Richtung des ersten SL beschleunigt wird. Da sich beide SL bewegen muss sich aber die Richtung der Beschleunigung ändern und diese Änderung wird mittels GW zwischen den SL übertragen (und auch in den restlichen Raum wodurch der Energieverlust zustande kommt). Ich bin mir hier nicht sicher, ob diese Richtungsänderung der Beschleunigung nicht doch ohne GW kommuniziert wird. Aber bei elektrischen Systemen ist es ja auch so, dass die Bewegungsänderung von Ladungsträgern per EM-Welle übertragen wird. Aber gitlt das auch schon im Nahfeld (ist es da nur ein Sonderfall von Welle oder kann man gar nicht von Welle reden und der Effekt wäre da was komplett anderes?) Das sind meine Verständnisschwierigkeiten. Hoffe, das ist trotz der späten Stunde noch verständlich

  68. #68 Torq
    6. April 2019

    @JoselB #67

    Wenn man die Bewegungen mittels ART beschreibt, sind GW nicht nötig. Sie entstehen als „Nebenprodukt“ der Bewegung durch die Raumzeit, und können diese durch den Energieverlust auch verändern und sich sogar gegeneinander beeinflussen, da sie die Raumzeit aufgrund ihrer Energie selbst verzerren.

    Es gibt aber in der SRT keine Notwendigkeit, innerhalb eines Gravitationsfeldes irgendwelche Informationen zu übertragen, damit ein Objekt (auch ein SL) sich auf seiner Bahn bewegt. Die Objekte folgen einfach ihrer Geodäten, d.h. sie befinden sich im freien Fall auf dem kürzesten Weg durch die Raumzeit. Es handelt sich also um einen rein geometrischen Effekt.

  69. #69 Torq
    6. April 2019

    @myself

    Immer ART, nicht SRT 😉

  70. #70 JoselB
    6. April 2019

    @Torq: Ich glaube wir reden etwas aneinander vorbei. Nehmen wir mal die Erde. Die Geodäte der Erde bildet im Raum näherungsweise einen Kreis mit einem Durchmesser von 2 AE. Dabei braucht eine Umrundung allerdings ein Jahr. Insgesamt bildet die Geodäte also eine Spirale mit einem Verhältnis vom Durchmesser zur Länge einer Wicklung von 2 AE / 1 LJ = 1,5 * 10⁻⁵. Das ist näherungsweise eine Gerade ohne Krümmung. Ein Körper der einer geraden Geodäte folgt hat keine Auswirkung auf das Gravitationsfeld und die Raumzeit ist näherungsweise statisch.

    Bei zwei SL kurz vorm Merger ist das Verhältnis aber eher bei 0,1 und höher. Hier ist die Krümmung nicht mehr zu vernachlässigen und es tritt ein Abberationseffekt auf, d.h. die Krümmung der Raumzeit ist einer ständigen Änderung durch die Bewegung der SL unterworfen. Diese Änderung breitet sich von den SL selbst mit Lichtgeschwindigkeit aus und bildet unter anderem die GW.

    Aber mit deiner Antwort hast du trotzdem meine Frage beantwortet. Da die äußere Krümmung und die Singularität eines SL beide der gleichen Geodäte folgen, bewegen sie sich synchron. Daran ändert sich auch nichts, wenn in Kombination mit dem externen G-Feld von dem anderen SL die äußere Krümmung selbst eine weitere Veränderung in der Raumkrümmung induziert (Nichtlinearität der ART) und so die SL aufeinander zu trudeln lässt (und GW abstrahlt).

    Von daher Vielen Dank!

  71. #71 Karl-Heinz
    6. April 2019

    @JoselB

    Die Raumzeit sei von mir aus flach.
    Wenn ein Körper sich konstant beschleunigt a(t) = konstant, so ist meine Frage jetzt, ob dieser Körper Gravitationswellen aussendet?

  72. #72 Torq
    6. April 2019

    Ein Körper der einer geraden Geodäte folgt hat keine Auswirkung auf das Gravitationsfeld und die Raumzeit ist näherungsweise statisch.

    Im euklidischen Raum nennt man Geodäten “Geraden”. Auch im Falle nicht-euklidischer Räume ist eine Geodäte die kürzeste Verbindung zwische zwei Punkten, z.B. das Teilstück eines Großkreises auf einer Kugelobertfläche (also auch der Erde). Das gilt auch im Falle der Raumzeit (die kürzeste Verbindung zwischen zwei Ereignissen ist in diesem Fall die mit der längsten Eigenzeit). D.h. jede Geodäte ist eine “gerade Geodäte”, unabhängig von der Krümmung.

    Was eine “statische Raumzeit” sein soll, verstehe ich jetzt leider nicht…

    Hier ist die Krümmung nicht mehr zu vernachlässigen

    Die Krümmung der Raumzeit ist nie zu vernachlässigen, sie bestimmt ja, wie die Geodäten verlaufen…

    und es tritt ein Abberationseffekt auf, d.h. die Krümmung der Raumzeit ist einer ständigen Änderung durch die Bewegung der SL unterworfen. Diese Änderung breitet sich von den SL selbst mit Lichtgeschwindigkeit aus und bildet unter anderem die GW.

    Hmm… Ich verstehe jetzt nicht, was die Abberation damit zu tun hat. Korrekt ist natürlich, dass die Krümmung der Raumzeit einer ständigen Änderung durch die Bewegung der SL unterworfen ist. Und dadurch entstehen dann auch Gravitationswellen.

  73. #73 Herr Senf
    6. April 2019

    #71 Frage – Antwort:
    jede beschleunigte Masse sendet Gravitationswellen aus ~m und ~a

  74. #74 JoselB
    6. April 2019

    @Karl-Heinz:
    Da bin ich mir nicht ganz sicher ob eine konstante Beschleunigung ausreicht. Ich meine mal gelesen zu haben, dass erst bei sich ändernden Beschleunigungen a'(t) != 0 Gravitationswellen entstehen. Macht für mich insofern Sinn, als dass erst dann eine sich ändernde Amplitude möglich ist, die für eine Welle ja notwendig ist. Bei einer kreisförmigen Geodäte im Raum, ist das wegen der Richtungsänderung auf jeden Fall gegeben.

  75. #75 JoselB
    6. April 2019

    @Torq
    Eine kreisförmige Geodäte im Raum, bzw Spirale in der Raumzeit wie die sich um die Sonne bewegende Erde ist aus sicht eines relativ zur Sonne ruhenden Beobachters beschleunigt. Daher ändert sich für diesen Beobachter auch das von der Erde ausgehende Gravitationsfeld. Diese Änderung erfolgt nicht instantan, ist aber nur durch die Beschleunigung (oder Änderung in der Beschleunigung, das ist einer der Punkte bei denen ich mir nicht sicher bin) verursacht und daher sehr gering. Auch hat das auf die Erde selbst erst mal keine weitere Auswirkung weil die Raumzeit dank Massenunterschied Erde/Sonne weiterhin annähernd statisch bleibt und damit auch die Geodäte unverändert bleibt. Ich denke in dem Punkt sind wir uns einig.

    Bei sich umkreisenden SL ist aber die Änderung der Gravitation durch das jeweils andere SL so gewaltig, dass sich die eigene Geodäte verändert was wiederum in geringerem Maße auf das andere SL zurück wirkt. Diese Änderungen entsprechen der Abberation. Ausschlaggebend ist hier nicht, dass jedes SL selbst auf einer Geodäte und damit aus eigener Sicht geradeaus fliegt, sondern dass vom jeweils anderen SL aus gesehen die SLs starken Beschleunigungen unterliegen bzw stark gekrümmte Geodäten besitzen.

    Mein ursprüngliches Problem war, dass ich nicht daran gedacht hatte, dass die äußere Krümmung der gleichen Geodäte wie die Singularität folgt (wenn man mal von Gezeitenkräften absieht. Die dürften die äußere Krümmung und damit auch den Ereignishorizont kurz vor dem Merger stark verzerren.)

  76. #76 Torq
    7. April 2019

    @JoselB

    Vorab: Ich bin hier kein Experte, begebe mich nachfolgend auf eher dünneres Eis, und Korrekturen sind sehr willkommen (Alderamin, Niels, Martin, das sind hier die Experten).

    Eine kreisförmige Geodäte im Raum, bzw Spirale in der Raumzeit wie die sich um die Sonne bewegende Erde ist aus sicht eines relativ zur Sonne ruhenden Beobachters beschleunigt. Daher ändert sich für diesen Beobachter auch das von der Erde ausgehende Gravitationsfeld. Diese Änderung erfolgt nicht instantan, ist aber nur durch die Beschleunigung (oder Änderung in der Beschleunigung, das ist einer der Punkte bei denen ich mir nicht sicher bin) verursacht und daher sehr gering. Auch hat das auf die Erde selbst erst mal keine weitere Auswirkung weil die Raumzeit dank Massenunterschied Erde/Sonne weiterhin annähernd statisch bleibt und damit auch die Geodäte unverändert bleibt. Ich denke in dem Punkt sind wir uns einig.

    Grundsätzlich ist es tatsächlich so, dass eine Bewegung entlang einer Geodäte durch eine gekrümmte Raumzeit von einem außenstehenden Beobachter als beschleunigte Bewegung wahrgenommen wird. Allerdings ändert sich für diesen Beobachter aus seiner Perspektive nicht einfach so die Geometrie der Raumzeit, dementsprechend auch nicht das Gravitationsfeld, so zumindest mein Verständnis.

    Das Gravitationsfeld ist eine andere Interpretation der ART, bei der die Raumzeit eine Minkowski-Raumzeit ist. Der metrische Tensor ist der Energie-Impuls-Tensor. Die Gravitationskraft stellt dabei eine Änderung der Zeitkomponente der Metrik da, Objekte fallen immer in Richtung der langsamerer laufenden Zeit. Die Metrik ist dabei genauso definiert, dass das Äquivalenzprinzip gilt. Das alles wird ausführlicher “nebenan” erklärt, oder auch in Martins Buch.

    Ich habe momentan irgendwie den Eindruck, dass du beide Konzepte zusammenwirfst, ohne dass mir genau klar ist, was du mir damit sagen willst… (Ich weiß z.B. noch immer nicht, was du mit statischer Raumzeit meinst…) Insofern bin ich mir nicht sicher, ob wir hier einer Meinung sind oder nicht…

    Bei sich umkreisenden SL ist aber die Änderung der Gravitation durch das jeweils andere SL so gewaltig, dass sich die eigene Geodäte verändert was wiederum in geringerem Maße auf das andere SL zurück wirkt. Diese Änderungen entsprechen der Abberation. Ausschlaggebend ist hier nicht, dass jedes SL selbst auf einer Geodäte und damit aus eigener Sicht geradeaus fliegt, sondern dass vom jeweils anderen SL aus gesehen die SLs starken Beschleunigungen unterliegen bzw stark gekrümmte Geodäten besitzen.
    Mein ursprüngliches Problem war, dass ich nicht daran gedacht hatte, dass die äußere Krümmung der gleichen Geodäte wie die Singularität folgt (wenn man mal von Gezeitenkräften absieht. Die dürften die äußere Krümmung und damit auch den Ereignishorizont kurz vor dem Merger stark verzerren.)

    Dazu kann ich nichts sagen. Zum einen siehe meine Ausführungen oben. Zum anderen reichen meine Mathematikkenntnisse bei weitem nicht aus, um auch nur ansatzweise zu begreifen, warum sich die beiden SL so verhalten, wie sie es tun. Das sind zwei Objekte mit extrem hoher Gravitation (um bei der Feld-Interpretation zu bleiben), die sich zudem extrem schnell bewegen, dazu mit ausgeprägten gegenseitigen Wechslwirkungen. Da benötigt man mehr als Papier und Bleistift, um sich einer Lösung der DGLen auch nur anzunähern… (Ich bin im Studium nicht mal wirklich bei Analysis 2 angekommen, gut dass ich den Schein nicht mehr gebraucht habe…)

  77. #77 bote19
    7. April 2019

    Torq,
    Der Name Schwarzes Loch impliziert irgendwie, dass es eine Kugelform hat.
    Zu der Eigenschaft von Kugeln gehört, dass sie nicht vollständig mit kleineren Kugeln gefüllt werden können, ohne eine Lücke zu hinterlassen.
    Im dreidimensionalen Raum kann eine zentrale Kugel höchstens 13 weitere Kugeln berühren, dabei bleiben aber Lücken.
    Im vierdimensionalen Raum ist die Anzahl 24, 25 oder 26. Ein Beweis fehlt. Leo Moser behauptet allerdings, dass sich die Kugeln dann durchdringen ohne eine Lücke zu hinterlassen.
    Meine Definition: Raum ist das, was zwischen der Materie ungefüllt bleibt. Die Materie selbst, die Atome liegen nicht dichtgepackt, was ja eine Dichte von Unendlich hätte.
    Aus: Martin Gardner, Mathematischer Zirkus.
    Das wäre dann doch der Beweis für die Behauptung von Alderamin, Masse benötigt keinen Raum mehr. Es bleibt im 4-dimensionalen Raum kein Raum mehr übrig.

  78. #78 Torq
    7. April 2019

    @bote19 #77

    Ich glaube nicht, da wir ja keinen vierdimensionalen Raum haben, sondern eine Raumzeit. Was mich an eine Diskussion mit Martin zum Vierervektor erinnert. Und das löst jetzt doch ein leicht ungutes Gefühl bei mir aus, da wir uns extrem vom Artikel gelöst haben und Alderamin nicht mehr mit diskutiert. (Solange er das getan hat, war es für mich wie oben gesagt ok.)

    Im Hinblick auf seine Anmerkung am Ende von #27: Wir sollten für eine solche Diskussion ggf. doch einen geeigneteren Ort suchen, z.B. nebenan… (Sein Blog, seine Regeln…)

    @Alderamin: Ich hoffe, mein Lob für den Artikel unter #25 ist nicht ganz unter gegangen. 🙂

  79. #79 Niels
    7. April 2019

    Tag zusammen.
    Ich gehe mal nur ganz kurz und allgemein auf ein paar Kernaspekte ein.

    Raumkrümmung wird verursacht durch Masse, Energiedichte, Druck, Scherung und Energiestromdichten (Energie-Impuls-Tensor). Irgendwas davon musst Du im Schwarzen Loch haben, sonst hast Du keine Schwerkraft. Beim Schwarzschild-Fall dürfte das ausschließlich Energiedichte sein, beim Kerr-Fall zusätzlich Energiestromdichte, aber frag’ lieber Martin Bäker oder Niels.

    Die Schwarzschild- und die Kerr-Löung sind Vakuumlösungen der ART. Hier hat also per Definition der Energie-Impuls-Tensor in jedem Punkt der Raumzeit den Wert Null.

    Das bedeutet aber nicht, dass diese Vakuum-Regionen keine Energie enthalten. Die Energie muss dann aber irgendwie in der Raumzeitkrümmung stecken, aber wie und wo sie lokalisiert ist, ist immer noch ungeklärt.

    Das ist die Sache mit der fehlenden Definition der Energie in der ART bzw. der Punkt mit der Frage der Energieerhaltung in der ART. Darüber haben wir schon mehrfach gesprochen.

    .
    Die Punkt- oder Ringsingularitäten sind, wie schon im Namen steckt, Singularitäten. 😉
    Hier ist die Metrik und damit auch die Raumzeit nicht mehr wohldefiniert, diese Bereiche gehören also rein mathematisch betrachtet gar nicht mehr zur Schwarzschild- bzw. Kerr-Lösung.

    Am Einfachsten sieht man das vielleicht daran, dass im Energie-Impuls-Tensor eben doch nicht einfach nur die Masse eingeht, sondern vielmehr die Energiedichte. Also die Energie pro Volumen. Über E=mc^2 kann man Masse problemlos in Energie umrechnen.
    Das Volumen der Singularität ist allerdings Null.
    Für die Energiedichte ergibt sich also unendlich.

    Genauso ergeben sich für Singularität dann auch für den metrischen Tensor bzw. die Krümmung Unendlichkeiten.
    Dieser Punkt (bzw. dieser Ring) lässt sich daher nicht sinnvoll als Teil der Raumzeit interpretieren und wird einfach “ausgeschnitten.”
    Wenn man über die Kerr- oder Schwarzschildraumzeit spricht, gehören die Singularitäten also gar nicht dazu.
    .

    @all
    Potentielle Energie, Gravitationsfeld, …
    Ihr vermischt fröhlich Newtonsche Gravitation und ART.
    In der ART gibt es kein Gravitationspotential, keine potentielle Energie und kein Gravitationsfeld.

    Diese Begriffe darf man also eigentlich nicht verwenden, wenn man ART-Effekte erklären will.

    Beschleunigung und Gravitationswellen
    Wenn sich etwas entlang einer Geodäte bewegt, ist dieses Objekt per Definition unbeschleunigt und strahlt auch keine Gravitationswellen ab.
    Man muss also auch mit dem Begriff der Beschleunigung sehr vorsichtig sein.

    Eine isolierte Massenverteilung strahlt genau dann Gravitationswellen ab, wenn die n-te zeitliche Ableitung des n-ten Multipolmoments des Energie-Impuls-Tensors dieser isolierten Massenverteilung nicht verschwindet, wobei das Multipolmoment mindestens das Quadrupolmoment sein muss.

    Bei elektromagnetischer Strahlung reicht es bekanntlich auch schon, wenn die erste Ableitung des Dipolmoments nicht Null ist.
    Das liegt daran, dass Ladungen positives und negatives Vorzeichen haben können, Massen aber nicht.

    Eine rotierende Scheibe oder eine sich um ihre Symmetrieachse drehende Hantel strahlen also keine Gravitationswellen ab.
    (Das gilt auch für einen sphärisch symmetrisch pulsierender Stern oder eine perfekt symmetrisch verlaufende Supernova.)

    Masse
    Ist euch wirklich klar, was dieser Begriff in der ART tatsächlich bedeutet?
    Das ist nämlich gar nicht so einfach.
    (Aber wenn man in der ART Probleme mit dem Energiebegriff hat, ist natürlich klar, dass das mit der Masse dann auch nicht so einfach sein kann.)

    Ganz grob vereinfacht muss Energie und damit äquivalente Masse wohl auch irgendwie in der Raumzeitkrümmung stecken.
    Das ist im wesentlichen die Ursache dafür, dass die Einsteingleichungen nichtlinear sind.
    “Gravitation” “erzeugt” sozusagen “Gravitation”.

    (Wir wissen tatsächlich schlicht nicht, was das genau bedeuten soll.)

    Als Masse M eines schwarzen Loches bezeichnet man jetzt die Masse, die ein Beobachter im Unendlichen wahrnehmen würde.

    Siehe etwa
    https://en.wikipedia.org/wiki/Mass_in_general_relativity

    und speziell für Schwarzschild und Kerr:
    https://en.wikipedia.org/wiki/Komar_mass
    Diese Vereinfachung gilt ausdrücklich nicht(!) für sich umkreisende Schwarze Löcher.

    Wegen der Aberration und dem “Herauswirken” der Gravitation über den Ereignishorizont lohnt es sich, die Wiki-Artikel “Speed of gravity” und “Aberration (Gravitation)” anzuschauen.

    Ganz grob zusammengefasst:
    Die Raumzeitkrümmung muss nicht aus dem schwarzen Loch kommen, sie ist schon draußen.
    (Okay, Gravitonen als Überträgerteilchen der Gravitation müssten das schon. Es ist ja kein Zufall, dass es mit einer Quantentheorie der Gravitation nicht klappt.)

    Energie [und damit letztlich auch Äquivalenzmasse] steckt nicht in der potentiellen Energie oder dem Gravitationsfeld (so etwas gibt es in der ART schließlich gar nicht), sondern kann nur in der Raumzeitkrümmung stecken.
    (Bei rotierenden schwarzen Löchern zusätzlich auch in der Rotation der Raumzeit.)

  80. #80 Torq
    7. April 2019

    Niels, vielen Dank!

    Einzig und alleine

    Potentielle Energie, Gravitationsfeld, …
    Ihr vermischt fröhlich Newtonsche Gravitation und ART.
    In der ART gibt es kein Gravitationspotential, keine potentielle Energie und kein Gravitationsfeld.

    Diese Begriffe darf man also eigentlich nicht verwenden, wenn man ART-Effekte erklären will.

    wundert mich… Siehe Bäker (Isaac) Seite 441 ff., dort geht es um die zwei Interpretationen der ART, geometrische Raumzeit vs. Gravitationsfeld im Minkowski-Raum.

  81. #81 Alderamin
    7. April 2019

    @Torq

    Und das löst jetzt doch ein leicht ungutes Gefühl bei mir aus, da wir uns extrem vom Artikel gelöst haben und Alderamin nicht mehr mit diskutiert. (Solange er das getan hat, war es für mich wie oben gesagt ok.)

    Ich habe im Moment einfach viel zu tun, das ist alles. Keine Zeit, mich da tief reinzudenken.

    Im Hinblick auf seine Anmerkung am Ende von #27: Wir sollten für eine solche Diskussion ggf. doch einen geeigneteren Ort suchen, z.B. nebenan… (Sein Blog, seine Regeln…)

    Solange ich nix sage, ist alles i.O. Diskutiert gerne weiter.

    @Alderamin: Ich hoffe, mein Lob für den Artikel unter #25 ist nicht ganz unter gegangen.

    Sorry, vielen Dank.

  82. #82 Niels
    7. April 2019

    @Torq

    Okay, das war nicht genau genug formuliert.

    Für statische (und mit etwas Anstrengung auch für stationäre) Raumzeiten kann man das so interpretieren.

    Bei sich eng umkreisenden schwarzen Löchern und den dabei auftretenden starken (nicht mehr in linearer Näherung betrachtbaren) Gravitationswellen geht das meiner Meinung nach nicht mehr.
    Und wenn man über Singularitäten sprechen will ebenfalls nicht.

  83. #83 Alderamin
    7. April 2019

    @bote19

    Das wäre dann doch der Beweis für die Behauptung von Alderamin, Masse benötigt keinen Raum mehr.

    Ich behaupte gar nicht, Masse brauche keinen Raum, ich sagte nur, die ART behauptet, dass die Masse zu einem Punkt (oder Ring) kollabiert, was aber vermutlich nicht die Realität beschreibt. Man weiß halt nicht, wie es im Inneren eines Schwarzen Lochs genau aussieht.

  84. #84 Alderamin
    7. April 2019

    @Niels

    Auch von mir danke. Ich komme später nochmal drauf zurück, im Moment keine Zeit.

  85. #85 Torq
    7. April 2019

    @Alderamin

    Alles klar, danke für deine Klarstellung.

    .
    @Niels

    Für statische (und mit etwas Anstrengung auch für stationäre) Raumzeiten kann man das so interpretieren.

    Bitte, bitte, erkläre mir doch mal jemand, was eine statische Raumzeit ist. Meine letzten beiden Fragen dazu waren nicht rhetorisch gemeint, ich weiß es einfach nicht (mehr?!). Ist das eine Raumzeit, in der der Energie-Impuls-Tensor in jedem Punkt der Raumzeit den Wert Null hat?

    Bei sich eng umkreisenden schwarzen Löchern und den dabei auftretenden starken (nicht mehr in linearer Näherung betrachtbaren) Gravitationswellen geht das meiner Meinung nach nicht mehr.

    Martin nennt beide Interpretationen „äquivalent“, von daher hatte ich ihn anders verstanden. Er schreibt aber auch, dass es bei hoher Gravitation und hohen Geschwindigkeiten schwierig wird. Aber ich denke, das gilt bei beiden Interpretationen. So oder so, mir ist das zu hoch, s. #76… Und ich denke nur Martin weiß, was Martin dazu denkt… 😉

    (Was so oder so Probleme mit sich bringt, ist sicher die Vermischung beider Interpretationen…)

    Und wenn man über Singularitäten sprechen will ebenfalls nicht.

    Ok, aber da versagt die ART ja generell, wie du auch selbst schreibst…

  86. #86 Niels
    8. April 2019

    @Torq

    Bitte, bitte, erkläre mir doch mal jemand, was eine statische Raumzeit ist.

    Eine Raumzeit ist stationär, wenn es ein Koordinatensystem gibt, in dem die Metrik-Koeffizienten zeitunabhängig sind.

    Sie ist statisch, wenn in dieser Metrik zusätzlich dann auch keine “Kreuzterme” wie dr*dt auftauchen.
    (Das Ganze ist dann also zeitsymmetrisch und zeittranslationsinvariant, es gibt also eine erhaltene “Energie”.
    In der ART “verallgemeinert” man das Noether-Theorem mit Hilfe sogenannter Killing-Vektorfelder. Killing-Vektorfelder beschreiben Symmetrieeigenschaften bzw. Isometrien der Raumzeit.
    Wenn eine Raumzeit ein oder mehrere sogenannte Killing-Vektorfelder besitzt, gibt es zu jedem dieser Killing-Vektorfelder jeweils eine Erhaltungsgröße der Raumzeit.
    Die saubere mathematische Definition ist leider ziemlich technisch und hat mit der Existenz bestimmter dieser Killing-Vektorfeld zu tun.
    [Stationäre Raumzeit besitzen ein zeitartiges Killingvektorfeld, bei statischen Raumzeiten ist dieses zeitartige Killingvektorfeld zusätzlich hyperflächenorthogonal.])

    Statische Raumzeiten sind daher per Definition immer auch stationär.

    Etwas anschaulicher:
    In einer stationären Raumzeit wie der Schwarzschild-Raumzeit gibt es Beobachter, die bei Experimenten, bei denen sie die Geometrie der Raumzeit (also die Krümmung) vermessen, zu allen Zeiten immer dasselbe Ergebnis erhalten.
    Das sind genau die Beobachter, deren Weltlinien in allen Schwarzschildkoordinaten außer der Schwarzschild-Zeitkoordinate t konstant sind. Also etwa Beobachter, die mit konstantem Radius um die Erde kreisen. Oder eben wir Menschen auf der Erdoberfläche.
    In einer statischen Raumzeit muss die Geometrie für diese Beobachter zusätzlich auch noch invariant unter Zeitumkehr sein. Dazu fällt mir leider nichts Anschauliches ein.

    .
    Nochmal anders und wahrscheinlich zu stark vereinfacht:
    Eine stationäre Raumzeit “ändert sich zeitlich nicht”, sie darf aber “rotieren”.
    Statische Raumzeit dürfen zusätzlich auch nicht “rotieren”.

    .

    Die bekannteste statische Raumzeit ist die Schwarzschild-Lösung für nicht-rotierende schwarze Löcher, die bekannteste stationäre Raumzeit ist die Kerr-Lösung für rotierende schwarze Löcher.
    Allgemeiner beschreibt die (äußere) Schwarzschild-Lösung / die Kerr-Lösung den “Außenraum” einer elektrisch neutralen, nicht-rotierenden / rotierenden Masse.

    Beispiele für dynamische Raumzeiten sind dagegen:
    A) Die FLRW-Lösung, die die Entwicklung des Universums beschreibt.
    B) Raumzeit mit Gravitationswellen gibt. Diese können grundsätzlich nie stationär sein.

  87. #87 Niels Geiger
    8. April 2019

    @Torq

    Und wenn man über Singularitäten sprechen will ebenfalls nicht.

    Ok, aber da versagt die ART ja generell, wie du auch selbst schreibst…

    Klar, aber man kann eben wenigstens darüber sprechen, was dort versagt, wie es versagt und an welcher Stelle es versagt.
    In Newton-Sprache wüsste ich nicht, wie man auch nur über die Existenz von Raumzeit-Singularitäten sprechen könnte?

  88. #88 Frank Wappler
    8. April 2019

    Niels schrieb (#86, 8. April 2019):
    > [Torq schrieb (#85, 7. April 2019): »Bitte, bitte, erkläre mir doch mal jemand, « …]
    > In einer stationären Raumzeit […] gibt es Beobachter, die bei Experimenten, bei denen sie die Geometrie der Raumzeit (also die Krümmung) vermessen, zu allen Zeiten immer dasselbe Ergebnis erhalten.

    Bitte, bitte, erkläre mir doch mal jemand, wie gegebene (denkbare, oder sogar auffindbare) Beobachter (d.h. insbesondere die von Einstein in Betracht gezogenen “materiellen Punkte” bzw. die von MTW erwähnten “principal identifiable points“) zumindest im gedanken-experimentellen Prinzip vorgehen sollten, um (Versuch für Versuch) die Geometrie der Raumzeit (also die Krümmung) zu vermessen.

  89. #89 Torq
    8. April 2019

    @Niels #86

    Erst Mal vielen, vielen Dank! Insgesamt bin ich deutlich schlauer als vorher, und die anschauliche Darstellung ist tatsächlich auch sehr anschaulich. 🙂

    (Dass die Schwarzschild-Lösung für nicht-rotierende schwarze Löcher statisch ist, das wusste ich, das sagt Dr. Google einem ja nach 5 Minuten. Aber was das im Allgemeinen bedeutet, davon hatte ich leider keinen Plan. Hoffe, im Bäker (2019) findet sich nix dazu, dann hätte ich ihn wohl doch nicht genau genug gelesen und bekomme am Ende noch von Martin auf die Finger… ;-))
    .
    @Niels #87

    Klar, aber man kann eben wenigstens darüber sprechen, was dort versagt, wie es versagt und an welcher Stelle es versagt.

    Da hast du natürlich recht…

    In Newton-Sprache wüsste ich nicht, wie man auch nur über die Existenz von Raumzeit-Singularitäten sprechen könnte?

    Da müsste jetzt Martin was zu sagen… Ich verstehe nur, dass die Begriffe dieselben wie bei Newton sind, dass sie aber anders hergeleitet werden, und wie das ganze grob funktioniert (s. #76). Wie weit die Äquivalenz effektiv geht, das weiß ich nicht. Und die Wahrscheinlichkeit ist groß, dass ich es nie vollständig verstehen werde… 😉

    Kennst du Martins Auisführungen dazu? (Ich habe im Blog jetzt auf die Schnelle nix gefunden, auch Dr. Google tut sich schwer, aber im Bäker (2019) wird es wie gesagt etwas ausführlicher behandelt…)

  90. #90 MartinB
    8. April 2019

    @alle
    Voll die Party hier und keiner sagt mir Bescheid?
    Ich gehe mal auf ein paar Punkte ein (und generell verweise ich darauf, dass das meiste auch in meinem Buch sehr detailliert erklärt wird…)
    https://scienceblogs.de/hier-wohnen-drachen/2019/01/29/hallo-isaac-mein-buch-ist-da/

    JoselB #16
    Richtig, im Zentrum z.B. der Erde hat man eine Zeitdilatation gegenüber einem entfernten Beobachter. Die Zeit verläuft da aber langsamer als überall drum herum, und da Objekte in Richtung langsam verlaufender Zeit fallen, ist man dort schwerelos.

    Alderamin #35 /Captain E#62
    Als Quelle für eine Raumzeitkrümmung dient immer der Energie-Impuls-Tensor. Aber auch in Bereichen, wo der Null ist, kann sich wie z.B. bei einer Gravitationswelle die Raumzeitkrümmung ausbreiten. Es ist deshalb bei einem Schwarzen Loch egal, was im Zentrum steckt, der Effekt außerhalb des SL kommt letztlich durch die Krümmung der Raumzeit am Ereignishorizont zu Stande.

    Soweit ich sehe, ist alles, was Captain E in #62 schreibt, richtig.

    @Karl-Heinz#72
    Soweit ich es weiß, erzeugt eine reine Beschleunigung keine Raumzeitkrümmung. Ein geladenes Teilchen (nur als Beispiel), das auf eine Ladung zufällt, krümmt die Raumzeit durch seinen Impulsstrom und seine Masse, aber soweit ich sehe hat die Änderung der Geschwindigkeit keinen direkten Einfluss und es entsteht keine Gravitationswelle. Gebe aber zu, dass ich nicht 100% sicher bin.

    @Niels
    Es ist ein bisschen unfair, der Feld-Formulierung der ART (mit Gravitationsfeld auf einer flachen Minkowski-Raumzeit) vorzuwerfen, sie könne nicht mit Singularitäten umgehen – kann die geometrische Formulierung ja auch nicht…
    Soweit ich sehe (siehe auch MIsnerThorneWheeler Kap 17 oder das Buch von Weinberg) kann man die gesamte ART als Feldtheorie auf einer flachen Raumzeit auffassen.

  91. #91 Torq
    8. April 2019

    @MartinB:

    Ich gehe mal auf ein paar Punkte ein (und generell verweise ich darauf, dass das meiste auch in meinem Buch sehr detailliert erklärt wird…)

    Du hast ja recht. Ich habe sogar im Zuge der Diskussion noch einige Male reingeschaut, z.B. zur Feldtheorie. (Und ich hoffe, dass sich Bäker (2019) durchsetzt.;-))

    Der Teufel steckt wie immer im Detail. Ich kenne das noch aus dem Studium, andere sicher auch (aus allen möglichen Lebenssituationen, in denen man was lernen will): Man liest ein Buch/Skript, blättert hin und her, liest es zwei/drei Mal und ist der festen Überzeugung, alles verstanden zu haben. Dann kommt eine konkrete Frage und man denkt sich – hä?!

    Wie war das noch mal? Habe ich dazu nicht was gelesen? Blätter, blätter, blätter, Stichwortverzeichnis, blätter, blätter, blätter. Oft hilft das, manchmal muss man sich weitere Literatur suchen, und manchmal kann man die spezielle Frage trotzdem nicht beantworten, egal was man tut…

    Da helfen im Studium dann die Tutoren (oder, mit sehr viel Glück, auch mal ein Assi oder – aber das ist praktisch schon unfassbar viel Glück – der Prof… Naja, die Mathematiker waren ganz nett und hilfsbereit…;-)).

    Hier helfen unsere Blogger und Spezialsten, mir ihrer unendlichen Geduld (außer bei RT-Leugnern…;->). Vielen Dank an euch alle!!!!!

  92. #92 Torq
    8. April 2019

    PS.: Heute ist “weitere Literatur suchen” natürlich meist ein Synonym für googeln… Ich war jetzt zu sehr in meiner Erinnerung verhaftet. Dafür galt damals noch “Don’t be evil”… 😉

  93. #93 MartinB
    8. April 2019

    @Torq
    War ja auch keine Kritik, sondern nur ein Versuch, etwas plumpe Werbung unterzubringen…

  94. #94 Torq
    8. April 2019

    @MartinB

    Deine Aussage hatte ich nicht als Kritik aufgefasst. Ich wollte eigentlich nur unterstreichen, dass das Buch prima ist, auch wenn evtl. nicht alles bei mir hängen geblieben ist. (Plump fand ich den Hinweis übrigens nicht.)

    Und bei dieser Gelegenheit wollte ich mich halt auch noch mal bei dir und den anderen Bloggern/Experten bedanken. 🙂

    (Immerhin habt ihr es geschafft, dass ich mich in dem ein oder anderen Forum schon als Experte fühle. Auch wenn sie es anders sehen, den meisten Einsteinkritikern kann man schon mit Grundwissen ganz gut begegnen… Allerdings meist ohne Effekt, wie schon häufiger von allen Seiten festgestellt, daher habe ich mittlerweile auch keinen Spaß mehr dran…)

  95. #95 Alderamin
    8. April 2019

    @MartinB

    Voll die Party hier und keiner sagt mir Bescheid?

    Hatte die Einladung zum Artikel doch auf Twitter verschickt, zweimal 😉

    Willkommen, Du hast uns noch gefehlt (echt jetzt!) 😉

  96. #96 MartinB
    8. April 2019

    @Alderamin
    Echt? Muss ich auf Twitter übersehen haben, sorry.

  97. #97 Herr Senf
    8. April 2019

    #90 MB “… erzeugt eine reine Beschleunigung keine Raumzeitkrümmung …”
    Na ja, nee, oder … doch?
    Masse krümmt die Raumzeit, die Raumzeit bestimmt die (ungestörte) Bewegung.
    Zeitliche (Zwangs/Kraft-)Änderungen erzeugen Störungen der Raumzeit-Metrik.
    Bei (egal wie) beschleunigten Massen nennt man diese Störungen Gravitationswellen.
    Wie schaffen wir die 100%? Grüße Dip

  98. #98 MartinB
    8. April 2019

    @HerrSenf
    Natürlich erzeugt eine bewegte Masse eine RZK, das ist ja klar. Aber eine mit konstanter Geschwindigkeit bewegte Masse erzeugt ja keine GW.
    Hier steht, dass die Bedingung komplizierter ist:
    https://physics.stackexchange.com/questions/256521/does-an-object-creates-gravitational-waves-when-only-accelerating-in-one-directi
    Hinzu kommt, dass man sich natürlich fragen muss, was die Beschleunigung verursacht. Zwei gegeneinander schwingende Massen zum Beispiel werden auch beschleunigt, erzeugen aber keine GW (weil sie kein Quadrupolmoment haben), wenn ich mich nicht ganz doll irre (steht glaube ich irgendwo im Misner Thorne Wheeler, den habe ich aber gerade nicht griffbereit):
    https://scienceblogs.de/hier-wohnen-drachen/2014/04/03/was-sind-gravitationswellen/?all=1

    Hier wird das auch diskutiert:
    https://www.physicsforums.com/threads/does-an-accelerated-particle-emit-gravitational-waves.533213/

    Bin leider gar nicht mehr geübt darin, Quadrupolmomente auszurechnen….

  99. #99 Karl-Heinz
    8. April 2019

    @Herr Senf

    MartinB, Niels und JoselB haben vollkommen recht. Nicht immer erzeugt eine Beschleunigung von Massen eine Gravitationswelle. Beschleunigung wäre die zweite Ableitung. Ist die Beschleunigung konstant ist die dritte Ableitung 0. Ich vermute, dass dann auch die dritte Ableitung vom Quadrupolmoments 0 ist.

    Es gilt:
    Die in den gesamten Raum pro Zeiteinheit abgestrahlte Energie hängt mit der dritten zeitlichen Ableitung des Massen-Quadrupolmoments zusammen.

  100. #100 MartinB
    8. April 2019

    @Karl-Heinz
    Sieht man das mit der 3. Ableitung des Quad-Moments irgendwie sofort oder ist das ne lange uninuitive Rechnung? Sollte nicht die 2. Ableitung des Quad-Moments relevant sein?

  101. #101 Herr Senf
    8. April 2019

    Schlagt mich 🙂
    Ein 100m-Läufer, der auf den ersten 10m beschleunigt, sendet GW aus.
    Kennt wer von den Spezialisten hier Prof Haas, wäre mal eine Anfrage

  102. #102 Niels
    8. April 2019

    @MartinB

    Ist auch die zweite Ableitung.

    Da war wahrscheinlich ich Schuld.
    Oben schrieb ich, die n-te zeitliche Ableitung des n-ten Multipolmoments darf nicht verschwinden.
    (Wobei das Multipolmoment mindestens das Quadrupolmoment sein muss.)

    Jetzt muss man sich daran erinnern, dass das Quadrupolmoment das zweite Multipolmoment ist und nicht das dritte.
    Das Monopolmoment ist nämlich der nullte Multipol, nicht der erste.

  103. #103 Karl-Heinz
    8. April 2019

    @MartinB

    Beim Dipolmoment ist die zweit Ableitung relevant bzw. die abgestrahlte Leistung ist zum Quadrat der zweiten Ableitung proportional.
    Nach Newton gilt aber F=m*a und actio gleich reactio (Impulserhaltung), weshalb m_1* a_1 + m_2*a_2 =F_1+F_2=0 folgt. Die Impulserhaltung ist also verantwortlich dafür, dass es keine gravitative Dipolstrahlung gibt. So stehts zumindest in einem Buch. Ich wasche meine Hände in Unschuld, bin aber bereit meinen Kopf darauf zu verwetten. 😉

  104. #104 MartinB
    8. April 2019

    @Herr Senf
    Ich denke, das muss so sein, der ist ja kurzfristig um freien Fall um die Erde und quasi wie ein Satellit.
    Wenn ich allerdings direkt senkrecht auf einen Planeten genau in Richtung Zentrum falle, sollte es meiner Ansicht nach keine GW geben (wegen der Symmetrie, der Planet fällt ja auch auf mich zu.)

    Die Frage ist ja, ob es auch GW gibt, wenn ich mit einer Rakete beschleunige oder ein geladenes Teilchen in einem Plattenkondensator.

    Die Glechungen stehen im MTW Kap. 36 (bin jetzt zuhause und kann nachgucken).

  105. #105 MartinB
    8. April 2019

    @Karl-heinz
    Das mit der Dipolstralung steht so im MTW (und auch in meinem Blogtext…), die Herleitung mit der 3. Ableitung ist im MTW, hab ich mir aber noch nicht im Detail angeguckt.
    Eigentlich will ich ja auch nur schnell wissen, ob ein teilchen, das sich auf ner geraden linie bewegt 8egal wie) überhaupt ein Quadrupolmoment haben kann…

  106. #106 MartinB
    8. April 2019

    @Niels
    Also, der MTW sagt in 36.1, dass im elektrischen Fall gilt, dass die abgestrahlte Leistung proportional zur zweiten Ableitung des Dipolmoments ist.
    Die zweite Abl. des Massendipolmoments verschwindet, wie Karl-Heinz ja auch schon schrub.

    Und die leistung eines Massenquadrupols ist laut MTW proportional zu I-dreipunkt ins Quadrat.

    So, und dann steht da, dass der spurfreie Anteil des zweiten Moments der Massenverteilung relevant ist (bzw. dessen 3. Ableitung). Vielleicht bin ich gerade zu doof für alles, aber ich sehe nach Gl 36.3 nicht, wie das jemals ungleich Null sein kann, wenn Teilchen sich entlang einer Linie bewegen.

    Im Fließbach sieht es ähnlich aus, wenn ich das auf die Schnelle richtig verstehe. Wobei alle diese Herleitungen immer periodische Prozesse annehmen und damit nicht so richtig zu einer beschleunigenden Rakete passen…

    Dieses paper scheitn zu sagen, dass eine Photonenrakete keine GW aussendet:
    https://arxiv.org/abs/gr-qc/9412063
    da steht aber
    “the gravitational wave amplitude generated by the energy-
    momentum of the photons exactly cancels the usual 1/r gravitational
    1wave amplitude generated by the accelerated motion of the rocket.”
    was ja heißen würde, dass im Allgemeinfall eine Rakete GW aussendet. (Das ist wohl Gl. 4.7 in dem paper.)

    Verwirrt-bin

  107. #107 Herr Senf
    8. April 2019

    Googelt man Quellen von GW gibt es “sowohl/als-auch”_Antworten.
    Aber eine kann wohl immer nur richtig sein, nicht 3 Bücher 5 Antworten.
    Meine Favoriten zum Streiten:
    pulsierende Kugel oder symmetrischer Kollaps – GW nein
    oszillierende oder “pulsierende” Hantel – GW nein
    Schwerpunktkollision unterschiedlicher Massen – tippe auf nein
    rotierende Kugel – GW ja
    rotierende Hantel (auch Stab oder Scheibe) – GW ja
    “einfach” beschleunigte Masse – GW ja, verliert Energie
    GW breiten sich nicht “auf” der Raumzeit aus, ändern die Krümmung “in”.

  108. #108 UMa
    8. April 2019

    @Herr Senf:
    rotierende (homogene) Kugel – GW nein

    Man sucht ja nach Gravitationswellen rotierender Neutronensterne. Die können aber nur auftreten, wenn es eine Elliptizität (oder Berge, außerhalb der Drehachse) gibt, so dass sich das Gravitationsfeld mit der Rotation ändert.
    Bisher wurde nichts gefunden, daher müssen die Neutronensterne schon sehr glatt sein, fast perfekte Kugeln, mit Ausnahme der Rotationsabplattung natürlich.

  109. #109 MartinB
    8. April 2019

    @Herr Senf
    Bis auf die einfach beschleunigte Masse sind die alle glaube ich unstrittig. Bei der Masse muss man eben immer auch fragen, wie die beschleunigt wird, oder nicht? Zwei Massen auf gerader Linie im freien Fal laufeinander zu bewegen sich ja erst mal wie die oszillierende Hantel.
    Hinzu kommt der Ärger, dass die Herleitungen ja eigentlich immer periodische Prozesse angucken.

    Ich hab jetzt mal geschummelt und nen echten Experten angefragt, mal sehen 😉

  110. #110 Karl-Heinz
    8. April 2019

    @MartinB

    Ich würde folgenden Ansatz verwenden.
    Zwei Teilchen mit Masse m_1 und m_2 auf einer Linie. Gesamtimpuls der beiden Teilchen ist konstant. Bewegung auf der Linie kann beliebig sein. Als ersten Fall würde ich m1 = m_2 und ihre Beschleunigung auf konstant setzen. Dann das ganze mal berechnen und gucken ob 3-te Ableitung null wird. Bin leider nicht zu hause um das selbst durchzurechn.

  111. #111 MartinB
    8. April 2019

    So ich habe jetzt Markus Pössel gefragt. Er sagt ganz klar, dass zwei gegeneinander schwingende Massen auch GW erzeugen können, nur halt keine Dipolwellen. Ein Quadrupolmoment haben die aber schon.
    hatte ich immer falsch verstanden, und leider steht das jetztbauchbfalschnin meinem Buch. Mist.

  112. #112 Herr Senf
    8. April 2019

    Hallo MartinB, nicht so schlimm, bin in #107 auch über ein falsches Uni-script gestolpert.
    Uma #108 hat ja die rotierende Kugel korrigiert, da ändert sich zeitlich das Feld nicht.
    Fazit: GW dann, wenn sich Konfiguration zeitlich ändert d³Q/dt³.
    eil-eil-eil … !
    Kleiner Trost S190408 ist da https://gracedb.ligo.org/superevents/S190408an/view/

  113. #113 Karl-Heinz
    9. April 2019

    @MartinB

    Falls du das Quadrupolmoment mal nachrechnest und grübelst. 😉

    ACHTUNG es sei darauf hingewiesen, dass manche Autoren das Quadrupolmoment anders definieren: also im Vergleich mit der Wikipedia Definition noch einen Faktor 1/3 hinein multiplizieren.

  114. #114 MartinB
    9. April 2019

    So jetzt habe ich das dank Markus’ Hilfe mit dem Quadrupolmoment auch verstanden: Ist mal wieder ganz einfach, wenn man drüber nachdenkt (ist schon ne Weile her, das ich sowas gemacht habe). Einen Quadrupol bekomme ich, wenn ich vier ladungen an den Ecken eines Quadrats habe, wobei immer gegenüberliegende Ladungen gleich und benachbarte entgegengesetzt sind. Hier grob skizziert, ich hoffe, die Faktoren stimmen:
    Nehmen wir an, die haben die Werte -1/2 und +1/2. Jetzt addiere ich nen Monopol dazu mit Stärke 2, dann erhöht sich der Wert jeder Ladung um 1/2. Also habe ich dann zwei Ladungen, die Null sind und zwei, die 1 sind. Mit anderen Worten, ich habe zwei gleiche Ladungen auf einer Linie, sonst nichts, also genau unsere Situation. Das zeigt wunderbar, dass die beiden ein Quadmoment haben (und damit, wenn sie schwingen, eine GW aussenden können) und dass sie kein Dipolmoment haben.
    Falls es mal ne Neuauflage von meinem Buch gibt, sollte ich das wohl im Anhang erklären, scheint ja eins von diesen Dingen zu sein, die nirgends stehen.

  115. #115 MartinB
    9. April 2019

    PS: wenn die massen schwingen, dann ist die dritte Ableitung (bei nem sinus) ja nicht Null, also gibt es GW.

  116. #116 Karl-Heinz
    9. April 2019

    @MartinB

    Wenn ich mich nicht verrechnet habe dann ist das Quadrupolmoment gleich m*d^2.
    m … Masse
    d … Abstand der Massen auf der Linie

    Annahme d(t) = d_0 + 2 (a*t^2)/2
    Beschleunigung a sei konstant. Die dritte Ableitung von (m*d(t)^2) ist demnach nicht 0 für alle t. Damit ergibt sich in diesem Fall auch eine Gravitationswelle, obwohl die Beschleunigung a konstant ist. Welch eine Überraschung.

  117. #117 MartinB
    9. April 2019

    @Karl-Heinz
    Sieht für mich erstmal richtig aus mit der 3. Ableitung.

  118. #118 JoselB
    9. April 2019

    @all
    Vielen Dank für die vielen Anregungen und Hinweise auf verwechselte Konzepte oder sonstige Ungenauigkeiten.
    @Torq #76
    Mit statischer Raumzeit meinte ich, dass alle zeitvarianten Anteile so klein ausfallen, dass sie näherungsweise vernachlässigt werden können. Ich hatte vereinfachend das Bild eines Partikels mit fast verschwindender Masse, der um einen massive sphärisch symmetrischen Himmelskörper kreist im Kopf als Gegenteil von zwei massiven sich eng umkreisenden Körpern. Hätte ich vielleicht dazu sagen sollen, das war so unsauber argumentiert.

  119. #119 JoselB
    9. April 2019

    @MartinB, Niels, Torq:
    Eine Frage stellt sich mir noch aus der Diskussion mit Torq. Das Quadrupolmoment benötigt eine Beschleunigung der Massen. Lokal sieht sich aber jede Masse die einer Geodäte folgt als unbeschleunigt. Gehe ich richtig in der Annahme, dass das Zustandekommen der GW abhängig vom Beobachter ist, d.h. ein Beobachter auf M1 sieht die Bewegung von M2 als Ursache der GW, ein Beobachter auf M2 wiederum die Bewegung von M1 und ein entfernter Beobachter sieht die Bewegung von M1 und M2 umeinander als Ursache. Oder ist die Sache noch komplexer? (M1 und M2 seien z.B. eng umkreisende Neutronensterne und der Beobachter idealisiert)

  120. #120 MartinB
    9. April 2019

    @JoselB
    Ich denke, das siehst du richtig. GWs hängen natürlich vom Beobachter ab. (Soweit ich mich entsinne, gilt das schon analog in der Elektrodynamik, eine beschleunigte Ladung sieht ihre eigene em-Welle auch anders.)

  121. #121 Karl-Heinz
    9. April 2019

    @JoselB

    Das Quadrupolmoment benötigt eine Beschleunigung der Massen. Lokal sieht sich aber jede Masse die einer Geodäte folgt als unbeschleunigt.

    Quadrupolmoment und Lokal, das passt irgendwie nicht zusammen. 😉

    Multipolentwicklung: Entwicklung des Potentials Φ( r ) einer Ladungsverteilung für große Entfernungen von der erzeugenden Ladungsverteilung ρ( r ) nach Potenzen von 1/r.

  122. #122 Klaus
    Laniakea-Haufen
    11. April 2019

    Jaja, immer feste druff auf die Ingenieure. Die dürfen sich dann das Hirn verknoten mit euren Ideen, die eingentlich funktionieren bis auf “ein paar kleinere ingenieurstechnische Probleme”.
    😉

  123. #123 UMa
    12. April 2019

    @Alderamin:
    Das mit dem Halo-Drive wird schwierig. Aufweitung des Strahls. Dann die notwendige Stärke. 1e10 W pro Tonne Masse um auch nur 0,08 m/s² zu erreichen. Das bedingt große Flächen, wie bei allen Lichtsegelantrieben.
    Stellare schwarze Löcher kann man vergessen. Er wird aber günstiger, je schwerer das schwarze Loch ist. Mit dem Milchstraßenzentrum könnte es eventuell gehen. Man könnte, startend aus 7500 AE Entfernung mit einem 1nm(!) Laser, mit einem wirklich gigantischen Lichtsegel auf etwa 0,044 c kommen, falls die Rotation schnell genug für die Verstärkung des Laser ist.
    Mit den Zentrum von M87 käme man auf relativistische Geschwindigkeiten, falls es die Rotation zulässt.

    Oder noch besser man nutzt den Jet aus. Dann wird man schnell genug, ist dann aber in der Richtung etwas eingeschränkt.

    Für stellare Objekte ist aber ein Lichtsegel, angetrieben von Lasern aus der Umlaufbahn eine Sterns, die ihre Energie aus der Sternenstrahlung beziehen, sehr viel einfacher. Und man kann es auch bei der Sonne anwenden und jedem anderen Stern.